Вы находитесь на странице: 1из 104

TOPNOTCH PRACTICE EXAM 1 FOR

MARCH 2020 and SEPT 2020


BATCHES
Всего 129/240

DISCLAIMER: This exam set is for practice purposes only. Your performance on this
exam doesn't guarantee success nor failure on your real exam. Use this exam to
help you identify your strengths and weaknesses. God bless!

Баллов: 0 из 0.

FULL NAME (Family Name, Given Name, M.I.) *

Andres, Jerome P.

STUDENT NUMBER *

1994

Ready? *

Yes

Biochemistry
Баллов: 10 из 20.

1. A premature 6 month old newborn is brought by his mother for


seizures. On examination you note an underweight infant with kinky,
brittle hair, who was hypotonic on physical examination. Ocular
examination was unremarkable. What is your consideration? *
1 из 1

A. Osteogenesis imperfecta

B. Alport Syndrome

D. Epidermolysis bullosa

E. Menkes Disease

Комментарий

The key information in establishing the diagnosis of this patient is the


genetic testing result, which is deficiency of b-galactosidase
leading to the accumulation of galactosylceramide. Hence the answer
is Krabbe Disease.
Fabry: a-galactosidase
Gaucher: B-Glucosidase
Farber: Ceramidase
Niemann Pick: Sphingomyelinase

2. A 36 year old female comes to you complaining of very sore gums,


loosening of her teeth and multiple petechial eruptions on her skin. You
adviser her to eat more of this vitamin, which you know aids in the
crosslinking of collagen fibers? *
1 из 1

A. Ascorbate
B. Biotin

C. Cobalamin

D. Cholecalciferol

E. Thiamine

Комментарий

Ascorbate or Vitamin C is responsible for collagen hydroxylation, and works


by acting as a coenzyme for proline and lysine hydroxylases in collagen
formation. Its deficiency presents as scurvy, which this patient seems to have.

3. This effect states that the release of oxygen from hemoglobin is


enhanced when the pH is low or when there is increased pCO2. *
0 из 1

A. Bohr Effect

B. Haldane Effect

C. Gibs-Donnan Effect

D. Tyndall effect

E. None of the above

Правильный ответ

D. Tyndall effect

Комментарий

The Bohr Effect refers to how oxygen behaves. Oxygen release from.
Hemoglobin is enhances in low pH states or in high pCO2 states.
The Haldane Effect refers to how CO2 behaves. The affinity of hemoglobin
and CO2 increases when O2 is less bound to hemoglobin.
The Gibbs Donnan effect refers to the behaviour of charged particles near a
semi-permeable membrane that sometimes fail to distribute evenly across
the two sides of the membrane. The Tyndall effect refers to scattering of light
in a colloid

4. An 32 year old male consults the ER due to signs of chronic liver failure
(ascites, jaundice, pruritus) with a noted wing-beating tremor and a dark
ring encircling the iris of both eyes. What mineral is known to cause this
disease? *
1 из 1

A. Iron

B. Zinc

C. Manganese
D. Copper

E. Selenium

Комментарий

Copper accumulation in the liver and brain causes the symptoms of Wilsons
disease (hepatolenticular degeneration). The Kayser-Fleischer rings due to
copper deposition in the eyes, is a pathognomonic sign.

5. In sickle cell disease, a change of the protein glutamate to valine


causes what change what kind of interaction among the amino acid
residues? *
0 из 1

A. Ionic

B. Hydrophilic

C. Hydrophobic

D. Covalent

E. Hydrostatic

Правильный ответ

C. Hydrophobic

Комментарий

Similar boards concept. A change in protein from glutamate (charged amino


acid prefering to be in the surface) to valine (a non polar amino acid
prefering the interior) leads to hydrophobic interactions and sickling.

6. A 5 year old boy was noted to have coarse facial features, abodminal
enlargement, behavioral disturbances and severe mental disability. On
physical examination, you note a clear cornea, and an obliterated
Traube's space. What is your consdideration? *
1 из 1

A. Hurler Syndrome

B. Hunter Syndrome

C. Sanfilippo Syndrome

D. Morquio Syndrome

E. Sly Syndrome
Комментарий

All these are mucopolysaccharidoses caused by the accumulation of


glycosaminoglycans normally degraded in lysosomes. Hunter is similar to
Hurler, but Hunter has no corneal clouding; both present with mental
retardation, coarse facial features and hepatosplenomegaly. There is no CNS
abnormality in Morquio syndrome.

7. In the well fed state, glycogen synthase is? *


0 из 1

A. Phosphorylated

B. Dephosphorylated

C. Inactivated

D. A and C

E. B and C

Правильный ответ

B. Dephosphorylated

Комментарий

In the well fed state dephosphorylated glycogen synthase is in an active form


to promote glycogenesis

8. This enzyme is important in the phosphorylation of glucose. It is


present in most tissues of the body and has a low Km and high affinity for
glucose? *
0 из 1

A. Glucokinase

B. Hexokinase

C. PFK-1

D. PFK-2

E. Puryvate kinase

Правильный ответ

B. Hexokinase

Комментарий

Hexokinase and glucokinase both are enzymes in the phosphorylation of


glucose to glucose 6. phosphate, however, hexokinase present in most
tissues while glucokinase is present only in the liver and pancreas.
Hexokinase has a lower Km and a higher affinity for glucose than
glucokinase, meaning hexokinase is activated first with lighter meals
(Skyflakes) for utilization of glucose in peripheral tissues, then eventually
glucokinase is activated to promote liver absorption, mostly for storage
(buffet)

9. The following molecules are donors of atoms needed for pyrimidine


synthesis EXCEPT? *
0 из 1

A. Glutamine

B. Aspartate

C. Glycine

D. Carbon dioxide

E. None of the above

Правильный ответ

C. Glycine

Комментарий

Donors of atoms in pyrimidine synthesis include glutamine, aspartate and


carbon dioxide only. Donors of atoms for purine synthesis include these
three, pkus glycine and N10-formyltetrahydrofolate.

10. What is the necessary components in the first step of heme


synthesis? *
1 из 1

A. Glycine

B. Succinyl-coA

C. Vitamin B6

D. All of the above

E. None of the above

Комментарий

Ochronisis or Alkaptonuria is a condition caused by absence of


Homogentisate oxidase, an enzyme of tyrosine metabolism that catalyzes the
conversion of homogentisate to maleylacetoacetate. Alkaptonuria typically
presents with chronic arthritis, brown-blue sclera, darkening of urine.

11. What is this vitamin which acts as a conenzyme in the transamination


process for amino acids? *
0 из 1

A. Thiamine
B. Riboflavin

C. Niacin

D. Pyridoxine

E. Cobalamin

Правильный ответ

D. Pyridoxine

Комментарий

Vitamin B6 or pyridoxine is a coenzyme for several reactions, including the


transamination of amino acids, decarboxylation of glutamate and histidine,
ALA synthase, cystathion synthase, glycogen phosphorylase, and the
synthesis of niacin from tryptophan

12. A 20 year old male patient complains of persistently loose fatty stools
for the past 3 months. What is the expected laboratory abnormality in
this patient? *
1 из 1

A. Anemia

B. Thrombocytopenia

C. Prolonged prothrombin time

D. Leukopenia

E. Basophilia

Комментарий

Similar board exams concept. Steatorrhea will cause malabsorption of your


fat soluble vitamins, namely Vitamin A, D, E and K. Vitamin K is related to
your clotting pathway, this the lab abnormality we expect from this is a
prolonged PT.

13. An infant is brought to the outpatient clinics by his father because of


noted sweet odor in his urine which he charaterized as pancake syrup
like. Aside from dietary restriction what vitamin may be supplemented to
promote more enzyme activity in some variants of this disease? *
0 из 1

A. Vitamin B1

B. Vitamin B2

C. Vitamin B5
D. Vitamin B6

E. Vitamin B7

Правильный ответ

A. Vitamin B1

Комментарий

Management of MSUD includes limitation of intake of valine, leucine and


isoleucine and for some thiamine dependent variants, large doses of Vitamin
B1/thiamine may be given to increase enzyme activity

14. Which is true of the malate aspartate shuttle? *


1 из 1

A. It is common in heart muscle and most tissues

B. It yields 1.5 ATPs per molecule of NADH

C. It is common in the brain tissues and white muscle

D. A and B

E. B and C

Комментарий

B and C describe the glycerophosphate shuttle The malate aspartate shuttle


yields 2.5 ATPs per NADH molecule

15. Which is NOT TRUE of the digestion and absorption of lipids? *


1 из 1

A. Bile salts are reabsorbed in the jejunum in the enterohepatic circulation

B. Pancreatic lipase requires colipase to function

C. After absorption, they are packaged as chylomicrons and released to the


lymphatics

D. Short and medium chain fatty acids are absorbed without the need for
micelles

E. Free fatty acids form mixed micelles to allow absorption when it comes into
contact with the enterocyte

Комментарий

Bile salts are reabsorbed for enterohepatic circulation, in the terminal ileum.
All others are true.
16. What is the mechanism of action of tetracycline? *
0 из 1

A. Binds to and inhibits bacterial DNA-dependent RNA polymerase

B. Interferes with movement of RNA polymerase

C. Interferes with initiation at the 30s subunit

D. Prevents binding at the A site of aminoacyl tRNA

E. Inhibits translocation at the 50s subunit

Правильный ответ

D. Prevents binding at the A site of aminoacyl tRNA

Комментарий

Tetracycline prevents binding of tRNA to the A site. Choice A refers to


rifampicin, choice B refers to dactinomycin, choice C refers to streptomycin
and aminoglycosides, choice E refers to clindamycin and macrolides.
Topnotch p. 46

17. A 3 week old neonate was noted to be floppy and hypotonic, with
reported diaphoresis on feeding and signs of heart failure. What is the
likely enzyme deficiency leading to this glycogen storage disease? *
0 из 1

A. Muscle phosphorylase

B. Branching enzyme

C. Debranching enzyme

D. Lysosonal acid maltase

E. Glucose-6-phosphatase

Правильный ответ

D. Lysosonal acid maltase

Комментарий

Lysosomal acid maltase deficiency or Type 2 Glycogen Storage Disease


(Pompe's) primarily causes glycogen accumulaation in heart muscle and a
cardiomyopathy with death from heart failure before 2 years old.

18. NADH and FADH2 is produced by steps catalyzed by the following


enzymes in the tricarboxylic acid cycle EXCEPT? *
1 из 1
A. Isocitrate dehydrogenase

B. Alpha-ketoglutarate dehydrogenase

C. Succinate dehydrogenase

D. Malate dehydrogenase

E. Glyceraldehyde 3-phosphate dehydrogenase

Комментарий

Glyceraldehyde 3 phosphate dehydrogenase also produces NADH but is part


of the Kreb's cycle. All others produce NADH/FADH2 in the citric acid cycle

19. Xeroderma pigmentosum is caused by which defect in DNA repair? *


0 из 1

A. Mismatch repair

B. Base excision repair

C. Nucleotide excision repair

D. Double strand break repair

E. Multifunctional repair

Правильный ответ

C. Nucleotide excision repair

Комментарий

Exposure to UV light causes damaged DNA with pyrmidine dimers (usually


thymine). UV specific endonucleases recognize this dimer and removes the
damaged strand, DNA polymerase synthesizes new DNA and ligase seals the
nick

20. This vitamin is a cofactor in the non-oxidative, reversible phase of the


Pentose Phosphate Pathway? *
1 из 1

A. Vitamin B1

B. Vitamin B2

C. Vitamin B6

D. Vitamin B9
E. Vitamin B12

Комментарий

Thiamine is a cofactor for the transketolases in the second phase of the


pentose phosphate pathway which yields ribose 5 phosphate

ANATOMY (21-40)
Баллов: 16 из 20.

21. The arch of the aorta comes from which aortic arch derivative? *
0 из 1

A. First Pair

B. Second Pair

C. Third Pair

D. Fourth Pair

Правильный ответ

D. Fourth Pair

Комментарий

See Topnotch p.21 handout. The fourth pair of aortic arch gives rise to the
arch of the aorta (left) and the right subclavian artery (right)

22. The primary muscle of inspiration is? *


1 из 1

A. Diaphragm

B. External intercostals

C. Internal intercostals

D. Subcostal

E. Innermost intercostal

Комментарий

The diaphragm is the primary muscle of inspiration.. Intercostal muscles


function to keep the intercostal spaces rigid, however the external
intercostals are more active during inspiration and the internal intercostals
are more active in expiration.

23. Housemaid's knee is a swelling of the bursa in what part of the knee?
*
1 из 1

A. Prepatellar

B. Suprapatellar

C. Infrapatellar

D. Pes anserinus

E. Popliteal

Комментарий

Prepatellar bursitis is caused by friction between the skin and the patella, or
from injury from compressive forces; manifesting as a swelling in the anterior
knee. This has been called housemaid's knee, however, other people who
work on their knees such as hardwood floor and rug installers may also
develop this.

24. The following describes the normal course of the ureter in females
EXCEPT: *
0 из 1

A. It crosses the bifurcation of the common iliac artery

B. Runs down the lateral wall of the pelvis behind the internal iliac artery

C. At the ischial spine it moves forward and medially beneath broad ligmaent

D. Passes inferiorly to the uterine vessels

E. Courses lateral to the uterine cervix

Правильный ответ

B. Runs down the lateral wall of the pelvis behind the internal iliac artery

Комментарий

The ureter runs down the lateral wall of the pelvis in front of the internal iliac
artery. All others are true

25. A 23 year old male was driving in a highway when he suffered a rear
end collision. What spinal ligament is likely to be affected from the crash
causing severe hyperextension of the neck? *
1 из 1

A. Anterior longitudinal ligmanet

B. Posterior longitudinal ligament


C. Ligamentum flavum

D. Interspinous ligament

E. Intertransverse ligament

Комментарий

Severe hyperextension of the neck (whiplash injury) occurs during the rear
end motor vehicle collisions especially when the head restraint is too low. In
these types of hyperextension injuries, the anterior longitudinal ligament is
severely stretched and may be torn. Similar board exam question.

26. This gait is seen among patients with a unilateral peroneal nerve
palsy causing weakness in foot dorsiflexion? *
1 из 1

A. Antalgic gait

B. Ataxic gait

C. Myopathic gait

D. Shuffling gait

E. Steppage gait

Комментарий

Steppage gait/neuropathic gait/equine gait is seen in patients with foot drop


(weakness of foot dorsiflexion), the cause of this gait is due to an attempt to
lift the leg high enough during walking so that the foot does not drag on the
floor. If unilateral, causes include peroneal nerve palsy and L5 radiculopathy.
If bilateral, causes include amyotrophic lateral sclerosis, Charcot-Marie-Tooth
disease and other peripheral neuropathies including those associated with
uncontrolled diabetes.

27. A newborn infant was born with transposition of the great vessels,
and was noted to be cyanotic with shortness of breath. Which structure is
essential for the infant's survival, that must remain patent while surgical
intervention has not yet been done? *
1 из 1

A. Sinus venosus

B. Ductus venosus

C. Ductus arteriosus

D. Umbilical arteries

E. Umbilical veins

Комментарий

The infant needs a patent right to left shunt that will allow mixing of
oxygenated and deoxygenated blood, without which an infant who has TGA
will not survIve

28. The following are retroperitoneal structures EXCEPT *


1 из 1

A. Adrenal glands

B. Thoracic portion of the esophagus

C. Ureters

D. Sigmoid colon

E. IVC

Комментарий

Mnemonic SADPUCKER: suprarenal gland (adrenals), aorta and IVC,


duodenum (2nd-4th part), pancreas (except tail), ureters, colon (ascending
and descending), kidneys, esophagus (thoracic), rectum (partial)

29. The following are contents of the carpal tunnel EXCEPT *


0 из 1

A. Flexor digitorum superficialis tendons

B. Flexor digitorum profundus tendons

C. Flexor pollicis longus tendon

D. Flexor carpi radialis tendon

E. Median nerve

Правильный ответ

D. Flexor carpi radialis tendon

Комментарий

The flexor carpi radialis tendon is not within the carpal tunnel. Contents of
the carpal tunnel include FDP (4), FDS (4) and FPL (1) tendons and the median
nerve

30. A physician is performing an annual physical examination to a 36


year old man for work clearance. Which part will he place his
stethoscope if he wants to auscultate the sounds of the pulmonic valve? *
1 из 1

A. Left 5th ICS MCL


B. 5th ICS left parasternal border

C. 2nd ICS left parasternal border

D. 2nd ICS right parasternal border

E. None of the above

Комментарий

Left 5th ICS MCL - MITRAL


5th ICS left parasternal border - TRICUSPID
2nd ICS left parasternal border - PULMONIC
2nd ICS right parasternal border - AORTIC

31. A 16 year old female is brought to you because of amenorrhea. On


examination she is of adequate height for age, there is normal growth of
breasts, axillary hair and pubic hair. On internal examination, vagina is
noted to be short, and an ultrasound reveals an absent uterus, vagina
and cervix. You do a karyotype which shows 46,XX. What does this
patient have? *
1 из 1

A. Turner Syndrome

B. Androgen Insensitivity Syndrome

C. Mayer Rokitansky Kuster Hauser Syndrome

D. Klinefelter Syndrome

E. None of the above

Комментарий

MRKH and AIS present very similar with absent upper vagina and uterus,
normal breast and possibly normal pubic hair development, however in
MRKH your gonads are ovaries, and karyotype i

32. A defect in the function of this conductive tissue may create the need
for an artificial cardiac pacemaker. What is this conductive tissue of the
heart? *
1 из 1

A. Atrioventricular bundle

B. Atrioventricular node

C. Purkinje Fiber

D. Moderator Band
E. Sinoatrial node

Комментарий

The SA node is the natural pacemaker of the heart.

33. Which layers of the adrenal medulla is responsible for production of


DHEA? *
0 из 1

A. Zona glomerulosa

B. Zona fascisulata

C. Zona reticularis

D. Medulla

E. None of the above

Правильный ответ

C. Zona reticularis

Комментарий

DHEA os dehydroepiandrosterone which is the precursor of androgens is the


primary hormone made by the zona reticularis
Glomerulosa-mineralocorticoids aldosterone, regulated by angiotensin II
Fasciculata - glucocorticoid cortisol, mediated by ACTH and CRH
Reticularis - androgens DHEA mediated by ACTH CRH
Chromaffin cells of medulla - catecholamines epinephrine, norepinephrine
mediated by preganglionic sympathetic fibers

34. Which is an acidophil? *


1 из 1

A. Follicle stimulating hormone

B. Prolactin

C. Luteinizing hormone

D. Thyroids timulating hormone

E. Adrenocorticotrophic hormone

Комментарий

All are basophils except prolactin and growth hormone

35. At what level does the azygos vein pierce the diaphragm? *
1 из 1

A. T8

B. T9

C. T10

D. T11

E. T12

Комментарий

Similar boards concept


Caval T8 - IVC, right phrenic nerve
Esophageal T10 - Esophagus, vagus nerve
Aortic T12 - Aorta, thoracic duct, azygos vein
Pi ate ten Evians at around 12: P, I 8 - 10 E VN, A T A 12

36. All of the cranial nerves below are involved in the swallowing process
EXCEPT: *
1 из 1

A. Musculocutaneous A. V

B. VII

C. VIII

D. X

E. XII

Комментарий

Cranial nerve VIII is a purely sensory nerve and does not assist in swallowing.
The other nerves facilitate swallowing.

37. Injury to this triangle of the neck causes drooping of the shoulder.
Identify the triangle. *
1 из 1

A. Occipital triangle

B. Supraclavicular triangle

C. Muscular triangle

D. Carotid triangle
E. Submandibular triangle

Комментарий

The occipital triangle contains the spinal accessory nerve CN 11, and lesions
in this triangle cause weakness of the trapezius with difficulty elevating the
scapula or shrugging the shoulder.

38. This zone of the liver is the first to be affected by ischemia *


1 из 1

A. Zone I

B. Zone II

C. Zone III

D. All of the above

E. None of the above

Комментарий

Zone I - periportal (portal vein) first affected by viral hepatitis, and ingested
toxins like cocaine
Zone II - yellow fever
Zone III - pericentral (centrilobular zone) first affected by ischemia, highest
concentration of CYP 450 enzymes, most senstive to metabolic toxins, site of
alcoholic hepatitis

39. An infant noted to have a difficult delivery had his arms adducted by
his side, medially rotated, extended and pronated, from weak deltoids,
supraspinaturs and infraspinatus, and biceps brachii muscles.. What is
this deficit called and what nerves are affected? *
1 из 1

A. Papal benediction sign. - Median nerve

B. Claw hand - Ulnar nerve

C. Saturday night palsy - Radial nerve

D. Erb palsy - C5-C6 roots

E. Klumpke palsy - C8-T1 roots

Комментарий

Erb palsy is caused by traction of the upper trunk at C5-C6 roots from lateral
traction of the neck on delivery of an infant, causing deficits in abduction,
lateral rotation and flexion supination of the involved side.

40. A 72 year old man walks in a stooped manner with the head and neck
forward, and with flexion at the knees, with little steps but with eventual
festination. What gait is this? *
1 из 1

A. Antalgic gait

B. Ataxic gait

C. Myopathic gait

D. Shuffling gait

E. Steppage gait

Комментарий

Shuffling/Parkinsonian gait occurs in patients with rigidity and bradykinesia.


Patient is stooped with the head and neck forward, with flexion at the knees.
The whole upper extremity is also in flexion with the fingers usually extended.
The patient walks with slow little steps known at marche a petits pas (walk of
little steps). Patient may also have difficulty initiating steps. The patient may
show an involuntary inclination to take accelerating steps, known as
festination.

MICROBIOLOGY (41-60)
Баллов: 15 из 20.

41. A 4 year old boy is brought to you for fever, abdominal pain and
occasional cough characterized as non-productive. He had a CBC done
from another hospital the day prior which showed leukocytosis with
predominance of eosinophilia. Which of the following choices is more
likely? *
1 из 1

A. URTI

B. Pulmonary tuberculosis

C. Ascariasis

D. Acute gastroenteritis

E. GERD

Комментарий

Similar boards concept


Causes of eosinophilia - PACCMAN
parasites, asthma, churg strauss syndrome, chronic adrenal insufficiency,
myeloproliferative disorders, allergic process, neoplasia (Hodgkins
lymphoma)

42. A 26 year old male comes to the ER weak with multiple episodes of
vomiting the previous day and voluminous rice water stools. He said he
drank water from an unknown source. Aside from replacement of fluid
losses, what is your antibiotic treatment of choice? *
1 из 1

A. Azithromycin
B. Cefazolin

C. Metronidazole

D. Penicillin

E. Rifampin

Комментарий

The antibiotic options for cholera include macrolides, fluoroquinolones, and


tetracyclines. The choice between them should be based on availability and
local resistance patterns.

43.Which of the following is characteristic of this gram negative bacteria


referred to in Case #42 *
1 из 1

A. Shooting star motility

B. Tumbling motility

C. Falling leaf motility

D. Swarming pattern

E. Fleur-de-lis pattern

Комментарий

Shooting star motility - Vibrio


Tumbling motility - Listeria
Falling lead motility - Giardia
Swarming pattern - Proteus
Fleur-de-lis pattern - Pseudomonas aeruginosa

44. What is your agar of choice if you wanted to culture this gram
negative bacteria in the laboratory? *
1 из 1

A. Charcoal-yeast extract

B. Barbour-Stoenner-Kelly

C. Thiosulfate Citrate Bile Salts

D. Skirrows

E. Thayer Martin
Комментарий

Charcoal-yeast extract - Legionella pneumophila


Barbour-Stoenner-Kelly - Borrelia burgdorferi
Thiosulfate Citrate Bile Salts - Vibrio cholera
Skirrows - Helicobacter, Campylobacter
Thayer Martin - Neisseria gonorrhea from non sterile sites

45. A 4 year old boy consults for appearance of initially bullous lesions on
the perioral area that eventually ruptured days later, leaving honey
colored crusts. What is the most common cause of this lesion? *
1 из 1

A. Herpes simplex virus Type 1

B. Hemophilus influenza

C. Neisseria gonorrhea

D. Group A beta hemolytic Streptococcus

E. Staphylococcus aureus

Комментарий

Honey-colored crusts are the characteristic lesions of impetigo. Impetigo is a


common skin infection most often caused by Staphylococcus aureus or
Streptococcus pyogenes.

46. After treatment of this boy's lesions, which of the following would be
an important diagostic test to check weeeks later for possible
complications? *
1 из 1

A. CBC

B. Platelet count

C. Liver function test

D. Urinalysis and serum creatinine

E. 2D-Echo

Комментарий

"We should send for urinalysis and creatinine to check for the kidney function
since impetigo caused by Streptococcus pyogenes (second most common
cause after Staphylococcus aureus)
Post strep GN can be caused by impetigo or strep throat, but Rheumatic fever
can only be caused by strep throat and not impetigo"

47. A pregnant female consults the OB Admitting Section for eruption of


painful erythematous vesicles on her left labia. She denies any vaginal
discharge or cervical tenderness or bleeding. What is the etiologic cause
of these lesions? *
1 из 1

A. Herpes simplex virus 2

B. Varicella zoster virus

C. Treponema pallidum

D. Chlamydia trachomatis

E. Human papilloma virus

Комментарий

HSV is a double stranded linear DNA virus. One of its subtype HSV2 is
associated with genital herpes, neonatal herpes (TORCH) and aseptic
meningitis. When latent, it stays in the lumbosacral ganglia.

48. What are the possible effects of this in the neonate if born via
spontaneous vaginal delivery EXCEPT? *
0 из 1

A. Neonatal meningitis

B. Eye pain and conjunctival erythema

C. Ulcerative lesions in the mouth, palate and tongue

D. Disseminated sepsis-like disease

E. All of the above

Правильный ответ

E. All of the above

Комментарий

Neonatal HSV may be classified into three main categories for therapeutic
and prognostic considerations: localized skin, eye, and mouth (SEM); CNS
with or without SEM; and disseminated disease. Both HSV-1 and HSV-2 may
cause SEM, CNS, or disseminated disease; however, HSV-2 has been
associated with a poorer outcome

49. Which of the following choices is the most common bacteria causing
osteomyelitis? *
0 из 1

A. Mycobacterium tuberculosis

B. Staphylococcus aureus

C. Streptococcus pyogenes
D. Enterococcus faecalis

E. Gram negative bacilli

Правильный ответ

B. Staphylococcus aureus

Комментарий

Staphylococcus aureus (including methicillin-resistant S. aureus), coagulase-


negative staphylococci, and aerobic gram-negative bacilli are the most
common organisms. In one study, staphylococci were isolated in 53 percent
of bone biopsy specimens, enterococci in 8 percent, streptococci in 12
percent, and anaerobes in 5 percent.

50. Which bone is most susceptible to spread of hematogenous


osteomyelitis? *
1 из 1

A. Appendicular long bones

B. Pelvis

C. Clavicle

D. Vertebrae

E. None of the above

Комментарий

"Vertebral osteomyelitis – Vertebral osteomyelitis is the most common form


of hematogenous osteomyelitis in adults
Sternoclavicular and pelvic osteomyelitis – After vertebral osteomyelitis, next
most common sites of hematogenous osteomyelitis in adults are the flat
bones of the axial skeleton (such as the sternoclavicular and pelvic bones)"

51. A 10 year old boy is brought by his grandmother to your clinic for
high grade fever, abdominal pain and a mucoid, bloody diarrhea with
associated tenesmus. Which is the most common cause of dysentery that
you have to consider in this case? *
1 из 1

A. EHEC (Entero hemorrhagic E. coli)

B. Campylobacter

C. Entamoeba

D. Salmonella

E. Shigella
Комментарий

Bacterialdysentery is caused by infection with bacteria from Shigella,


Campylobacter, Salmonella, or enterohemorrhagic E. coli. Diarrhea from
Shigella is also known as shigellosis. Shigellosis is the most common type of
dysentery, with about 500,000 cases diagnosed in the United States each
year.

52. Which of the following are features found in the causative agent in
#51 *
0 из 1

A. Comma-shaped, microaerophilic, growns on Skirrow’s agar

B. Motile, oxidase negative, H2S producer

C. Urease positive, oxidase negative, H2S producer

D. Obligate aerobe, oxidase positive, H2S non-producer

E. Non-motile, oxidase negative, H2S non-producer

Правильный ответ

E. Non-motile, oxidase negative, H2S non-producer

Комментарий

"Choice A is Campylobacter
Choice B is Salmonella
Choice C is Proteus
Choice D is Pseudomonas aeruginosa"

53. Which of the following is known to cause a false positive in a VDRL


test? *
0 из 1

A. Viral infection EBV

B. Rheumatic fever

C. Lupus

D. Leprosy

E. All of the above

Правильный ответ

A. Viral infection EBV

Комментарий

BDRL checks for a non specific antibody that reacts with beef cardiolipin, it is
sensitive but not specific; thus it can be falsely positive in pregnancy, viral
infections like EBV and hepatitis, drugs, rheumatic fever and lupus/leprosy
54. A 32 year old female comes to your clinic for a 3 day history purulent
discharge from her vagina canal with associated dysuria and
dyspareunia. She reports having unprotected intercourse with multiple
male partners. On examination there is noted yellowish thick discharge in
the canal and tenderness on bimanual examination. Which virulence
factor of this bacteria is responsible for its antigenic variation? *
1 из 1

A. Lipooligosaccharide

B. Opa protein

C. Outer membrane porins

D. Pili

E. Capsule

Комментарий

The pili of Neisseria gonorrhea promotes adherence to epithelial cells and


antigenic variation to the bacteria, allowing for multiplle repeated infections.

55. The agar of choice to culture the bacteria in #54 is Thayer Martin
agar. This selective medium contains the following antibiotics EXCEPT: *
1 из 1

A. Vancomycin

B. Nystatin

C. Colistin

D. Bacitractin

E. None of the above

Комментарий

Thayer Martin agar or VCN contains antibiotics which prevent overgrowth of


other endogenous bacteria. Vancomycin targets other gram positive
bacteria, colistin targets gram negative bacteria and nystatin targets most
fungi

56. A 12 year old girl developed rheumatic fever with symptoms of fever
38.7 degrees, migratory polyarthritis, an evanescent rash with ring
margins and subcutaneous nodules, however no carditis was
documented on echocardiography. For how long should she receive
prophylaxis in the form of pencillin G IM injections? *
0 из 1

A. For 5 years
B. Until 21 years old

C. For 10 years

D. Until 40 years old

E. Lifelong

Правильный ответ

B. Until 21 years old

Комментарий

For rheumatic fever without carditis, prophylaxis is usually 5 years or until 21


years old, whichever is longer
For RF with carditis but no valvular heart disease, prophylaxis is 10 years or
well into adulthood
For RF with residual valvular heart disease, prophylaxis is 10 years since last
episode, 40 years of age or lifelong

57. An immunocompormised patient with AIDS was admitted at the ER


for cough and diffculty of breathing. He was also noted to be persistently
hypoxemic on room air. Stat CXR done showed diffuse bilateral ground
glass opacities with pneumatoceles. Bronchial washings were positive for
Pneumocystis pneumonia. What is your drug of choice? *
1 из 1

A. Co-amoxiclav

B. Fluconazole

C. Trimethroprim-sulfamethoxazole

D. Amphotericin B

E. Metronidazole

Комментарий

The drug of choice for PCP for treatment and prophylaxis is cotrmoxazole.
Prophylaxis is usually intitiated at a CD4 count<200.

58. Which of the following features may be used to distinguish Taenia


solium from Taenia saginata EXCEPT? *
1 из 1

A. Proglottids

B. Rostellum

C. Scolices
D. Intermediate host

E. None of the above

Комментарий

The proglottids and scolices of T. solium and T. saginata are morphologically


distinguishable and can be used to make a species diagnosis. T. solium
proglotids have <10 branches while T. saginata have >12. They also both
have pigs as intermediate host

59. What is the drug of choice for Taenia infection *


1 из 1

A. Mebendazole

B. Praziquantel

C. Ivermectin

D. Pyrantel pamoate

E. Metronidazole

Комментарий

Praziquantel is the drug of choice for Taenia solium and saginata

60. What is the reservoir hosts of the Ebola virus? *


1 из 1

A. Bats

B. Rodents

C. Swine

D. Cattle

E. Dogs

Комментарий

It is thought that fruit bats of the Pteropodidae family are natural Ebola virus
hosts. Ebola is introduced into the human population through close contact
with the blood, secretions, organs or other bodily fluids of infected animals
such as fruit bats, chimpanzees, gorillas, monkeys, forest antelope or
porcupines found ill or dead or in the rainforest.

Physiology (61-80)
Баллов: 8 из 20.
61. Which of the following is an example of a primary active transport? *
0 из 1

A. Na-glucose cotransport in the intestines

B. Na-Ca exchanger in the heart

C. H-K ATPase in the stomach

D. Glucose transport in muscle

E. Na-K-2Cl transporter in the thick ascending limb

Правильный ответ

C. H-K ATPase in the stomach

Комментарий

Primary active transport - occurs against an electrochemical gradient,


requires direct input of metabolic energy in the form of ATP; Ca-ATPase in
the sarcoplasmic reticulum, H-K ATPase in the gastric parietal cells, Na-K
ATPase in most cell membranes

62. Phenylalanine derivatives include the following EXCEPT? *


0 из 1

A. Melanin

B. Thyroxine

C. Levodopa

D. Melatonin

E. Norepinephrine

Правильный ответ

D. Melatonin

Комментарий

Phenylalanine Derivatives: tyrosine, dopamine, norepinephrine, epinephrine,


T3/T4, melanin, levodopa

63. Solution A and B are separated by a membrane permeable to a


divalent cation. The concentration of the cation in Solution A is 20mM and
the concentraion in solution B is 200mM. Assuming that the
electrochemical gradient of this divalent cation is 60mV, electrochemical
equilibrium is achieved is Solution A is at: *
1 из 1

A. -30mV
B. 0mV

C. +30mV

D. +60mV

E. +90mV

Комментарий

E=-60mV/2 * log (1/10) = -30*-1 = 30mV

64. Which of the following is correctly paired? *


1 из 1

A. Hypertropia - light focuses in front of the retina - need convex lens

B. Myopia - light focuses behind retina - need concave lenses

C. Astigmatism - irregular lens curvature - need cylindric lenses

D. Presbyopia - loss of accommodaion - need concave lenses

E. None of the above

Комментарий

"Hypertropia/farsighted - light focuses behind retina, need convex lens


Myopia/near sighted - light focuses in front of retina, need concave lenses
Presbyopia - loss of accommodation, need convex lens"

65. When comparing CSF with blood concentrations, all of the following
are lower in CSF than in blood EXCEPT? *
0 из 1

A. Glucose

B. Ca

C. K

D. Creatinine

E. Cholesterol

Правильный ответ

D. Creatinine
Комментарий

CSF magnesium and creatinine is higher than in the blood

66. Increased lipolysis of fat cells is mediated by which receptor? *


0 из 1

A. Alpha 1

B. Alpha 2

C. Beta 1

D. Beta 2

E. M 3

Правильный ответ

C. Beta 1

Комментарий

B1 - increased renin secretion of kidney, increased lipoylsis of fat cells, heart


rate, contractility

67. Increased aldosterone by the adrenal cortex results to which of the


following? *
1 из 1

A. Hypokalemia

B. Alkalosis

C. Hypernatremia

D. None of the above

E. All of the above

Комментарий

Increased aldosterone increases Na reabsorption, and the function of the Na-


H exchange and Na-K exchange. It also increases thirst and bauses
vasoconstriction, increasing mean arterial pressure back to normal

68. This phase of the cardiac cycle is known as the longest phase? *
0 из 1
A. Reduced ventricular filling

B. Rapid ventricular filling

C. Isovolumetric ventricular relaxation

D. Reduced ventricular ejection

E. Rapid ventricular ejection

Правильный ответ

A. Reduced ventricular filling

Комментарий

The phase of reduced ventricular filling or diastasis is the longest phase of


the cardiac cycle where ventricular filling continues but at a slower rate. It is
dependent on heart rate.

69. This phase of the SA nodal action potential is called the slow
depolarization phase via If channels that accounts for the automaticity of
the SA node? *
0 из 1

A. Phase 0

B. Phase 1

C. Phase 2

D. Phase 3

E. Phase 4

Правильный ответ

E. Phase 4

Комментарий

There are no phase 1 and 2 for SA node action potential. Phase 4 is


characterized by slow depolarization caused by an increase in Na
conductance which results in an inward Na currect via If, accounting for the
automaticity of the pacemaker activity of the heart

70. The following will shift the O2-hemoglobin dissociation curve to the
right EXCEPT? *
1 из 1

A. Hypercarbia

B. High temperatures
C. Low pH

D. Decrease in 2,3 DPG

E. Exercise

Комментарий

Increase in 2,3DPG will shift the curve to the right

71. Which is the major site of airway resistance? *


0 из 1

A. Trachea

B. Main bronchus

C. Medium sized bronchi

D. Bronchiole

E. Alveoli

Правильный ответ

C. Medium sized bronchi

Комментарий

The major site of airway resistance is medium sized bronchi. The smallest
airways would seem to offer the highest resistance but they do not because
of their parallel arrangement.

72. Nitrate derivatives, sulfonamides and other drugs are implicated in


methemoglobinemia. What is the pathophysiology of
methemoglobinemia? *
1 из 1

A. Base pair change leading to red blood cells with deformed shape

B. Ferrous in hemoglobin is converted to ferric

C. Hemolysis of red blood cells from oxidative stress

D. Defect in spectrin and ankyrin

E. Ferric in hemoglobin is converted to ferrous

Комментарий

A is sickle cell disease, C is G6PD, D is hereditary spherocytosis


73. Which substance is both filtered and secreted by the renal tubules
and is used to measure renal plasma flow? *
0 из 1

A. Creatinine

B. Evans blue

C. Tritiated H2O

D. Inulin

E. Para-aminohippuric acid

Правильный ответ

E. Para-aminohippuric acid

Комментарий

"Para-aminohippuric acid used to measure RPF


Inulin used to measure GFR
Tritiated H2O to measure TBW
Evans blue and RISA to measure plasma"

74. Which of the following is FALSE? *


0 из 1

A. Parietal cells in the antrum secrete HCl

B. Chief cells in the body and fundus secrete pepsinogen

C. G cells in the antrum secrete gastrin

D. Parietal cells in the body and fundus secrete intrinsic factor

E. Mucous cells in the antrum secrete mucous and pepsinogen

Правильный ответ

A. Parietal cells in the antrum secrete HCl

Комментарий

Similar boards question. Gastric parietal cells are found in the body and
funds of the stomach than in the antrum. They secrete both HCl and intrinsic
factor

75. When fertilization occurs, the corpus luteum does not degenerate
and instead secretes this substance which helps maintain the initial
stages of pregnancy by promoting further decidualization of the uterus *
1 из 1

A. Human chorionic gonadrotropin

B. Human placental lactogen

C. Estrogen

D. Progesterone

E. Testosterone

Комментарий

The corpus luteum secretes progesterone, which is a steroid


hormone responsible for the decidualization of the endometrium (its
development and maintenance of pregnancy). It also secretes minimal
amounts of estradiol and inhibin.

76. In the peripheral tissues, 80% of T3 is produced by deiodination of T4.


The following are true about Type II deiodinase EXCEPT *
0 из 1

A. It is the predominant deiodinating enzyme in the muscle, brain, pituitary, skin


and placenta.

B. It has a low Km for T4

C. It is inhibited by PTU

D. It prefers to deiodinate T4 more than reverse T3

E. It is a T4-5'-deiodinase

Правильный ответ

C. It is inhibited by PTU

Комментарий

"The two types of T4-5'-deiodinase (types I and II) are distinguished by their
location, biochemical properties, and responses to physiologic stimuli
●Type I is the predominant deiodinating enzyme in the liver, kidney, and
thyroid. It has a high Km for T4, is propylthiouracil (PTU) sensitive, and
deiodinates in the following order: rT3>T4>T3.
●Type II is the predominant deiodinating enzyme in muscle, brain, pituitary,
skin, and placenta. It has a low Km for T4, is not inhibited by PTU, and
deiodinates T4>rT3."

77. Major form of CO2 transport in the blood is? *


…/1

A. Dissolved carbon dioxide

B. Carbaminohemoglobin
C. HCO3-

D. Carboxyhemoglobin

E. None of the above

Нет правильных ответов

78. The following causes a shift of K into the cell causing hypokalemia
EXCEPT? *
1 из 1

A. Insulin

B. B-adrenergic agonists

C. Alkalosis

D. Hypoosmolarity

E. Digitalis

Комментарий

Digitalis inhibits the Na-K ATPase which brings Na out of the cell and K into
the cell. Thus digitalis increases K outside the cell, causing hyperkalemia.

79. GnRH is normally secreted in a pulsatile manner in the hypothalamus.


However, if GnRH were to be secreted continuously, what is the expected
effect on LH and FSH? *
1 из 1

A. LH will increase and FSH will decrease

B. FSH will increase and LH will decrease

C. Both will not be affected

D. Both will increase

E. Both will decrease

Комментарий

A continuous secretion of GnRH will cause a decrease in LH and FSH levels.


This is how GnRH analogues after the initial flare, can still be used to
antagonize reproductive axis resulting eventually to hypoestrogenis,

80. Metabolic alkalosis is caused by which of the following? *


0 из 1

A. Diarrhea

B. Thiazide diuretics

C. Renal tubular acidosis

D. Salicylate intoxication

E. High altitude

Правильный ответ

B. Thiazide diuretics

Комментарий

Due to the mechanism of contraction alkalosis, thiazide and loop diuretics


can cause metabolic alkalosis due to an upregulated RAAS and increase in
aldosterone that promotes H+ excretion

Legal Med (81-100)


Баллов: 15 из 20.

81. According to the Dangerous Drugs Act, which of the following will
qualify for life imprisonment as penalty? *
0 из 1

A. 4 grams of opium

B. 6 grams of morphine

C. 8 grams of cocaine

D. 20 grams of marijuana resin

E. 400 grams of marijuana

Правильный ответ

D. 20 grams of marijuana resin

Комментарий

Similar boards concept. Possession of 10 grams or more of marijuana resin,


or 500 grams or more of marijuana will qualify for lifetime imprisonment

82. What is the blood alcohol level at which a person has disorientation,
mental confusion, sensory disturbances and slurred speech? *
1 из 1

A. 50mg %
B. 100mg %

C. 150-300mg%

D. 300mg %

E. 400mg %

Комментарий

Review blood alcohol levels. At 150-300mg%, clinical signs and symptoms


include disorientation, mental confusion, decreased sense of pain, sensory
disturbances, slurred speech and exaggerated emotions

83. Cocaine is classified as what kind of drug? *


1 из 1

A. Hypnotic

B. Sedative

C. Hallucinogen

D. Stimulant

E. None of the above

Комментарий

Stimulant - cocaine, amphetamine; Hallucinogen - Amanita mushroom,


hemp, marijuana, LSD; Sedative.- barbiturates, methalqualone; Hypnotic -
opiates

84. A 72 year old female patient was diagnosed with Stage III breast
cancer. After explaining the therapeutic options, and their respective
benefits and risks, the patient decides no further treatment. What
bioethics principle is applicable in this case? *
1 из 1

A. Self-determination

B. Non-maleficence

C. Justice

D. Beneficence

E. None of the above


Комментарий

Review 4 bioethical principles. Recurring concept in the boards.

85. You are a newly graduated resident about to set up your own
practice You plan on making signboards at the front of your clinic. You
know that the signboard shall not exceed what size? *
1 из 1

A. 1x1 meters

B. 1x2 meters

C. 2x2 meters

D. 2x3 meters

E. 3x4 meters

Комментарий

A physician shall not employ agents in the solicitation and recruitment of


patients. For the promotion of medical practice, a physician may use
professional cards, classified advertising, publications, internet, directories
and signboards. Signboards shall not exceed one by two (1x2) meters in size.

86. A dead baby was found in the toilet. On examination, the baby was
noted to have its first upper molars erupted. What is the approximate
age of the baby? *
0 из 1

A. 6 months

B. 9 months

C. 12 months

D. 18 months

E. 24 months

Правильный ответ

D. 18 months

Комментарий

Upper molars - 13-19 months

87. Level of evidence needed for convicion of a criminal case? *


1 из 1
A. Guilt beyond reasonable doubt

B. Preponderance of evidence

C. Substantial evidence

D. Witness statement

E. None of the above

Комментарий

"Criminal - guilt beyond reasonable doubt


Civil - preponderance of evidence
Administrative - substantial evidence"

88. Dr. X runs his outpatient clinics and gets 1000 pesos for every patient
he sends for a CT scan at Diagnostic Lab Y. What kind of fee is this? *
1 из 1

A. Simple contractual fee

B. Retainer fee

C. Contingent fee

D. Dichotomous fee

E. Straight fee

Комментарий

Dichotomous fee (or commission or fee splitting) is unethical but legal and
involves sharing a fee with another physician, lab, or drug company not
based on services performed

89. As a new resident, while doing your chart rounds to your admitted
patients, you make sure to meticulously perform your history and PE, and
carefully review your diagnostic and therapeutic orders, so that you do
not miss anything that could lead to negligent care of your patients. What
bioethics principle is applicable in this case? *
1 из 1

A. Self-determination

B. Non-maleficence

C. Justice

D. Beneficence
E. None of the above

Комментарий

Review 4 bioethical principles. Recurring concept in the boards.

90. Components of the Harvard criteria for brain death EXCEPT? *


1 из 1

A. Deep coma and no withdrawal to painful stimuli

B. Flat electroencephalogram

C. Apnea for 3 minutes off a ventilator

D. Drugs, hypothermia and other causes excluded

E. Test repeated in 48 hours

Комментарий

Repeat above evaluation in 24 hours

91. Which of the following qualifies as illegal practice of medicine? *


1 из 1

A. A registered optometrist fitting prescription lenses on a patient

B. A registered dentist performing a tooth extraction

C. A nurse who takes the patient's blood pressure and announces the result

D. A mother giving a household remedy to her child

E. A second year medical student signs his name with an MD at the end, on all his
documents

Комментарий

Topnotch handout page 17

92. Conclusive evidence of sex? *


1 из 1

A. Voice and manner of speech

B. Muscular development and fat distribution


C. Presence of ovary in female and testis in male

D. Presence of vagina, uterus and accessories in female, penis in make

E. Presence of developed and large breasts in female

Комментарий

Topnotch handout page 28

93. If a person dies of a dangerous communicable disease, how many


years later can a permit to disinter be secured? *
0 из 1

A. 1 year

B. 3 years

C. 5 years

D. 7 years

E. 10 years

Правильный ответ

C. 5 years

Комментарий

"Non-dangerous communicable disease - 3 years


Dangerous communicable disease - 5 years"

94. Patient Z suffered from an injury that incapacitated him and required
medical attendance for 15 days. What is the classification of his injury? *
0 из 1

A. Mutilation

B. Serious physical injury

C. Less serious physical injury

D. Slight physical injury

E. Physical injurt inflicted in tumultous affray

Правильный ответ
C. Less serious physical injury

Комментарий

Less serious physical injury - period of medical attendance >10 but <30 days

95. Mr. Y reports a desire to continue using Drug A, however he reports


that he has the capacity to physically refrain from using it if needed; what
is this condition? *
1 из 1

A. Drug addiction

B. Drug habituation

C. Drug abuse

D. Drug withdrawal

E. None of the above

Комментарий

Drug habituation - the desire to use the drug is not compulsive, and is merely
psychical; with no tendency to increase the dose; and with the capacity to
refrain physically from using it

96. Characteristtics of an exit gunshot wound EXCEPT? *


1 из 1

A. Everted

B. Stellate

C. No tattooing or smudging

D. With contact ring

E. Paraffin test negative

Комментарий

Exit wound - always bigger, everted, can be stellate, no contusion collar, no


tattooing or smudging, underlying tissues seen protruding from wound,
always negative paraffin test

97. The ER resident knows that the supply of medical equipment in the
hospital is limited however he does not let sex, age, race, religious belief
or sexual orientation get in the way of his patients equal opportunities to
receive their needed care. What bioethics principle is applicable in this
case? *
1 из 1
A. Autonomy

B. Non-maleficence

C. Justice

D. Beneficence

E. None of the above

Комментарий

Review 4 bioethical principles. Recurring concept in the boards.

98. The following are true for HIV post exposure prophylaxis for
healthcare personnels EXCEPT *
1 из 1

A. PEP should be initiated as soon as possible (1-2 hours)

B. 72 hours post exposure, PEP is less likely to be effective

C. PEP is given as a single antiretroviral drug

D. Recommended duration is four weeks or until testing shows source patient is


HIV negative

E. Monitoring for CBC, renal and liver function should be done while on PEP

Комментарий

We administer a three-drug regimen for all HCP who opt for post-exposure
prophylaxis (PEP).

99. You are in a wide field playing with your friends when suddenly a
thunderstorm happend and lightining strikes. You are caught outdoors.
What is the best option? *
1 из 1

A. Lie down on the ground

B. Stand beside a tree

C. Crouch close to the ground

D. Gather together as a group with your friends

E. Go near a source of water


Комментарий

Crouch close to the ground. All the others are not advised and may put one at
even greater risk of injury

100. Level of evidence needed for convicion of a administrative case? *


0 из 1

A. Guilt beyond reasonable doubt

B. Preponderance of evidence

C. Substantial evidence

D. Witness statement

E. None of the above

Правильный ответ

C. Substantial evidence

Комментарий

"Criminal - guilt beyond reasonable doubt


Civil - preponderance of evidence
Administrative - substantial evidence"

Pathology (101-120)
Баллов: 9 из 20.

101. During your Pathology rotation, you are given a slide with neurons
containing PAS-positive cytoplasmic inclusions which your resident said
are Bunina bodies. These bodies are found in which disorder? *
1 из 1

A. Amyotrophic Lateral Sclerosis

B. Friedrich Ataxia

C. Huntington Disease

D. Rabies

E. Spinal Muscular Atrophy

Комментарий

Bunina bodies are remnants of autophagic vacuoles seen in ALS with the
degeneration of UMN and LMN.

102. Which is the first and primary receptor that HIV attaches to? *
0 из 1
A. CD4

B. CCR5

C. CXCR4

D. APOBEC3G

E. NF-KB

Правильный ответ

A. CD4

Комментарий

Robbins p. 246 - HIV infects cells by using the CD4 molecule as a receptor
and various chemokine receptors as coreceptors.

103. A 28 year old primigravid is on her 3yth week AOG of pregnancy.


What is the most significant adaptation or change thought to occur in her
uterus which allowed her to accommodate the increasing size of her
term infant? *
0 из 1

A. Endometrial hyperplasia

B. Endometrial hypertrophy

C. Myometrial hyperplasia

D. Myometrial hypertrophy

E. All of the above

Правильный ответ

E. All of the above

Комментарий

An increase in size of myometrial smooth muscle cells is responsible for the


increase in uterine size in pregnancy. Endometrial hyperplasia also occurs
however it is less significant than myometrial hypertrophy.

104. Which is NOT true of Phyllodes tumors? *


1 из 1

A. Phyllodes tumors arise from intralobular stroma

B. Phyllodes tumors usually present in the 6th decade of life


C. Phyllodes tumors are characterized as having higher cellularity, mitotic rate,
nuclear polymorphism, stromal overgrowth and infiltrative borders

D. Most Phyllodes tumors are high grade lesions

E. Phyllodes tumors recur locally but do not metastasize

Комментарий

Phyllodes tumors are low-grade

105. What is the management of Phyllodes tumors? *


0 из 1

A. Wide excision or mastectomy

B. Wide excision or mastectomy with sxillary lymph node dissection

C. Wide excision + chemotherapy

D. Wide exciion + radiotherapy

E. Observation only

Правильный ответ

A. Wide excision or mastectomy

Комментарий

Tumors often recur locally unless they are treated with wide excision or
mastectomy. Regardless of grade, lymphatic spread is rare and axillary
lymph node dissection is contraindicated

106. This peptide synthesized in the liver is a regulator if iron absorption


in the gut by acting on enterocytes and duodenal cells? *
1 из 1

A. Ferritin

B. Transferrin

C. Hepcidin

D. Hemoglobin

E. Albumin

Комментарий
Iron absorption is regulated by hepcidin, a small peptide from the liver that
binds to ferroportin causing ferroportin endocytosis and degradation. With a
decline in ferroportin, iron becomes trapped in duodenal cells as mucosal
ferritin.

107. This is a premalignant lesion in sun damaged skin or in skin exposed


to ionizing radiation, hydrocarbons or arsenicals? *
1 из 1

A. Seborrheic keratoses

B. Actinic keratosis

C. Squamous cell carcinoma

D. Basal cell carcinoma

E. Acanthosis nigricans

Комментарий

Actinic keratosis are tan brown, red or skin colored rough sandpaperlike
lesions less than 1cm in diameter that are classified as premalignant in sun
damaged skin.

108. The following are true regarding the pathology of ALS except? *
0 из 1

A. There is loss of upper motor neurons in the cerebral cortex

B. There is loss of lower motor neurons in the spinal cord and brainstem

C. The anterior roots of the spinal cord are thin and the precentral motor gyrus
atrophic

D. There is a reduction of anterior horn neurons with associated reactive gliosis

E. Remaining neurons gave a PAS-negative cytoplasmic inclusions

Правильный ответ

E. Remaining neurons gave a PAS-negative cytoplasmic inclusions

Комментарий

Remaining neurons have PAS positive inclusions called Bunina bodies

109. Inactivation of PIGA gene which is crucial for enzymes needed in the
synthesis of membrane associated complement regulatory proteins can
be found in what disease entity? *
0 из 1
A. Hereditary Spherocytosis

B. Sickle Cell Anemia

C. Thrombotic thrombocytopenia purpura

D. Hemolytic uremic syndrome

E. Paroxysmal nocturnal hemoglobinuria

Правильный ответ

E. Paroxysmal nocturnal hemoglobinuria

Комментарий

PIGA gene mutations lead to a deficiency of three GPI-linked proteins(CD55,


CD59 and C8 binding protein), and when red cells are deficient in these
factors, they are abnormally susceptible to lysis and injury by the C5b-C9
membrane attack complex, causing paroxysmal nocturnal hemoglobinuria

110. A week old newborn male infant is brought by his mother to you for
findings of irritability, poor feeding, a periumbilical redness with note of a
hyperemic fleshy mass seen midline, below the umbilicus. What is your
consideration *
1 из 1

A. Patent urachus

B. Urachal cyst

C. Exstrophy of the bladder

D. Meckels diverticulum

E. Exophytic papilloma

Комментарий

Exstrophy of the bladder is a developmental failure of the anterior abdominal


wall of the abdomen and the bladder, so that the bladder communicates
directly through a large defect or lies in an open sac The exposed bladder
mucosa may undergo colonic glandular metaplasia and is subject to
infections and should undergo surgical correction

111. A 37 year old male is brought to you with findings of fever,


headache, aphasia and visual changes and tea-colored urine. On workup,
patient had low hemoglobin at 100mg/dl with schistocytes on peripheral
blood smear, and a low platelet count at 80,000 per microliter. PT/PTT
are normal.What is your primary consideration? *
0 из 1

A. Hereditary Hemorrhagic Telangiectasia

B. Immune Thrombocytopenic Purpura


C. Hemolytic Uremic Syndrome

D. Thrombotic Thrombocytopenic Purpura

E. Paroxysmal Nocturnal Hemoglobinuria

Правильный ответ

D. Thrombotic Thrombocytopenic Purpura

Комментарий

"Similar boards concept. Patients with thrombotic thrombocytopenic purpura


typically report an acute or subacute onset of the following symptoms related
to central nervous system (CNS) dysfunction, anemia, or thrombocytopenia:
Neurologic manifestations include alteration in mental status, seizures,
hemiplegia, paresthesias, visual disturbance, and aphasia
Fatigue may accompany the anemia
Severe bleeding from thrombocytopenia is unusual, although petechiae are
common. Clinical manifestations may also include feverthat occurs in
approximately 50% of patients and a dark urine from hemoglobinuria."

112. This condition occurs when large destructive pituitary adenomas


develop after surgical removal of the adrenal glands due to the loss of
the inhibitory effect of adrenal corticosteroids on a preexisting
corticotroph adenoma? *
1 из 1

A. Cushing syndrome

B. Sheehan syndrome

C. Ashermann syndrome

D. Nelson syndrome

E. Turcot Syndrome

Комментарий

In Nelson syndrome, because adrenals are absent, hypercortisolism does not


develop and patients present usually due to the mass effects of the pituitary
tumor (Robbins p. 1079)

113. The following characterize a Nectrotic type of cell death EXCEPT? *


1 из 1

A. Swelling of cell with increase in size

B. Fragmentation of nucelus to nucleosome sized fragments

C. Ongoing adjacent inflammation

D. Disruption of the plasma membrane


E. Enzymatic digestion of cellular contents with leaking out

Комментарий

Topnotch handout page 1

114. During recruitment of leukocytes to the site of inflammation what


adhesion molecule is responsible for diapedesis? *
0 из 1

A. CD31

B. IL-8

C. integrins

D. LTB4

E. selectins

Правильный ответ

A. CD31

Комментарий

CD31 or PECAM-1 is the adhesion molecule that assists in the migration of


adherent leukocytes across the endothelium

115. Which of the following associated cancers is correctly paired with its
etiologic agent? *
0 из 1

A. Hepatocellular carcinoma - Clonorchis sinensis

B. Squamous cell carcinoma - HPV 6, 11

C. Bladder SCCA - Schistosoma japonicum

D. Kaposi Sarcoma - HHV 6

E. Adult T-cell leukemia - HTLV-1

Правильный ответ

E. Adult T-cell leukemia - HTLV-1

Комментарий

Clonorchis sinensis and Opistorchis viverrini is associated with


Cholangiocarcinoma; HPV 16, 18, 31, 33 is associated with Squamous cell
CA; Schistosoma hematobium is associated with bladder SCCA and HHV 8
(KSHV) is associated with Kaposi Sarcoma

116. This type of RNA does not encode proteins, but rather modulates
the translation of target mRNAs, and is responsible for posttranscriptional
silencing of the expression of some genes. *
0 из 1

A. tRNA

B. miRNA

C. lncRNA

D. siRNA

E. mRNA

Правильный ответ

B. miRNA

Комментарий

Similar board exam question. Page 4 Robbins

117. Which of the following is an example of a T-cell mediated


hypersensitivity reaction? *
1 из 1

A. Bronchial asthma

B. Arthus reaction

C. Psoriasis

D. Serum sickness

E. Acute rheumatic fever

Комментарий

Examples of Type IV hypersensitivity reaction includes Rheumatoid arthritis,


Multiple sclerosis, Type 1 DM, IBS, Psoriasis and Contact sensitivity

118. Which of the following is the known cause of primary achalasia? *


0 из 1

A. Trypanosoma cruzi infection

B. Diabetic neuropathy
C. Amyloidosis

D. Down Syndrome

E. Unknown

Другое:

Правильный ответ

E. Unknown

Комментарий

The cause of primary achalasia from ganglion cell degeneration is unknown.


The other choices are causes of secondary achalasia.

119. A 35 year old male consults you for persistent nasal pus-like
drainage with crusting, nasal stuffiness, and sinus infections,
nasopharyngeal erosions, as well as a persistent cough, fever, fatigue
and bloody urine. He denies any history of asthma, allergic rhinitis, or
connective tissue disorders. Which marker of disease activity will you
request? *
0 из 1

A. PR3-ANCA

B. MPO-ANCA

C. ANA

D. anti-Scl 70

E. anti-dsDNA

Правильный ответ

A. PR3-ANCA

Комментарий

Wegeners granulomatosis or granulomatosis with polyangitis is characterized


by necrotizing granulomas of the upper respiratory tract, granulomatous
vasculiyid snd glomerulonephritis. PR3-ANCA or c-ANCA is present in 95% of
cases and is a useful marker of disease activity.

120. The following factors are implicated in gastropathy EXCEPT? *


1 из 1

A. Alcohol

B. Bile
C. Non-steroidal anti-inflammatory drugs

D. Stress

E. Yogurt

Комментарий

Gastritis is a mucosal inflammatory process. If neutrophils are present it is


termed gastritis, however if inflammatory cells are rare or absent, the term
gastropathy is used. Agents that cause gastropathy include NSAIDs, alcohol,
bile and stress

Pharmacology (121-140)
Баллов: 6 из 20.

121. This dopaminergic pathway is the one implicated for the


extrapyramidal symptoms caused by typical antipsychotic *
1 из 1

A. Mesolimbic

B. Mesocortical

C. Nigrostriatal

D. Tuberoinfundibular

E. None of the above

Комментарий

Similar board exam concept. The nigrostriatal pathway is involved in motor


planning, dopaminergic neurons stimulate purposeful movement.
Antagonism of this leads to extrapyramidal symptoms. The mesolimbic
pathway is associated with the positive symptoms of schizophrenia. The
mesocortical pathway is associated with negative symptoms of schizophenia.
The tuberoinfundibular pathway regulates prolactin release

122. Alternative to DEC for treatment of filariasis EXCEPT? *


1 из 1

A. Doxycycline

B. Albendazole

C. Ivermectin

D. Praziquantel

E. None of the above


Комментарий

Similar boards concept. Other alternatives to filariasis treatment include


doxycyline, albendazole or ivermectin however some of these target only
microfilaria OR macrofilaria, and may need repeated dosing

123. A 36 year old man was diagnosed with Hypertension Stage I and
was started with an antihypertensive. He comes back to you on follow up
complaining of depressed mood, fatigue and moments of anhedonia.
What antihypertensive may possibly be causing this? *
0 из 1

A. Reserpine

B. Metoprolol

C. Enalapril

D. Indapamide

E. Amlodipine

Правильный ответ

A. Reserpine

Комментарий

Side effects of reserpine include sedation, severe psychiatric depression,


suicidal ideation and rebound suicide

124. A 24 year old female is brought to the emergency room for


ingestion of multiple tablets 1 hour ago. As part of the toxicology team,
you consider giving activated charcoal. Activated charcoal is effective for
the following drugs EXCEPT? *
1 из 1

A. Iron

B. Barbiturates

C. Carbamazepine

D. Tricyclic antidepressant

E. Valproic Acid

Комментарий

Activated charcoal is not effective for iron, lithium, potasium, alcohols,


cyanide, corrosives, acids etc.

125. Arrhythmogenesis and toxicity with digoxin is worsened by *


0 из 1

A. Hyperkalemia

B. Hypermagnesemia

C. Hypercalcemia

D. Hyponatremia

E. Hypophosphatemia

Правильный ответ

C. Hypercalcemia

Комментарий

Digitalis toxicity is increased by hypokalemia, hypomagnesemia and


hypercalcemia. Hypokalemia may be induced by loop diuretics and thiazide
use.

126. What is the antifungal that works by inhibiting fungal squalene


oxidase and is used for dermatophytosis and onychomycosis? *
0 из 1

A. Azole

B. Allylamine

C. Echinocandin

D. Polyene

E. None of the above

Правильный ответ

B. Allylamine

Комментарий

Allylamine is the drug class of terbinafine, butenafine and naftifine which are
all squalene oxidase inhibitors.

127. A 20 year old female aplastic anemia patient has had persistently
low platelet counts. The hematologist considers giving a hematopoetic
growth factor to promote platelet production. Which can they give? *
0 из 1

A. Darbepoetin alpha

B. Filgrastim
C. Plerixafor

D. Leucovorin

E. Eltrombopag

Правильный ответ

E. Eltrombopag

Комментарий

Eltrombopag, oprelvekin, thrombopoetin or romiplastim are all


megakaryocyte growth factor given subcutaneously that promotes platelet
production.

128. All of the following are GnRH analogues that can cause tumor flare
up in the first few weeks of therapy EXCEPT? *
0 из 1

A. Cetrorelix

B. Goserelin

C. Leuprolide

D. Nafarelin

E. Triptorelin

Правильный ответ

E. Triptorelin

Комментарий

Cetrorelix, ganirelix, abarelix and degarelix are all GnRH antagonists, thus
they do not cause tumor flare up when used for advanced prostate CA.

129. In the scenario above, what drug may be given to prevent acute
flare ups of prostate cancer with treatment of GnRH analogues? *
1 из 1

A. Flutamide

B. Fluvestrant

C. Aminogluthetimide

D. Metyrapone
E. Mifepristone

Комментарий

Flutamide, bicalutamine and nilutamide are an andrgen antagonist given


with GnRH analogs to prevent acute flare up of prostate cancer.

130. The following cephalosporins are capable of crossing the BBB


particularly in inflamed states EXCEPT? *
0 из 1

A. Cefoperazone

B. Cefuroxime

C. Cefotaxime

D. Ceftazidime

E. Ceftriaxone

Правильный ответ

A. Cefoperazone

Комментарий

Cefoperazone has poor BBB penetration.

131. Sumatriptan, the drug of choice for acute migraine, exerts its action
on what receptor? *
0 из 1

A. 5HT2 antagonist

B. 5HT3 agonist

C. 5HT3 antagonist

D. 5HT1D antagonist

E. 5HT1D agonist

Правильный ответ

E. 5HT1D agonist

Комментарий

Sumatriptan is a 5HT1D recetor agonist that causes vasoconstriction and


modulates neurotransmitter release. The 5HT3 antagonist is ondansetron.
The 5HT2 antagonist include ergonovine and ergotamine.
132. This DMARD inhibits interleukin 1 and interleukin 2 productiion and
secondarily inhibits T-cell responsiveness *
0 из 1

A. Azathioprine

B. Chloroquine

C. Mycophenolate Mofetil

D. Cyclosporine

E. Sulfasalazine

Правильный ответ

D. Cyclosporine

Комментарий

Cyclosporine is a DMARD used for RA, SLE, polymyositis, dermatomyositis


and other autoimmune diseases, acting by inhibiting IL-1 and IL-2 receptors.

133. In cases of ethylene glycol poisoning, we usually give fomepizole as


treatment in order to slow down the formation of what compound? *
0 из 1

A. Formaldehyde

B. Oxalic acid

C. Formic acid

D. Sulfuric acid

E. Nitric acid

Правильный ответ

B. Oxalic acid

Комментарий

Ethylene glycol causes severe acidosis and renal damage when it is


converted to oxalic acid by alcohol dehydrogenase.

134. This drug is an ADH antagonist to V2 receptors, inhibiting


reabsorption of free water *
1 из 1
A. Indapamide

B. Furosemide

C. Metolazone

D. Tolvaptan

E. Desmopressin

Комментарий

Tolvaptan, conivaptan and lixivaptan are ADH antagonist aquaretic drugs


used for congestive heart failure, cirrhosis and SIADH.

135. This drug class works I both the central and peripheral nervous
system in order to prevent degrataion of levodopa, by inhibiting catechol-
O-methyltransferase. *
0 из 1

A. Selegiline

B. Tolcapone

C. Pramipexole

D. Bromocriptine

E. Carbidopa

Правильный ответ

B. Tolcapone

Комментарий

Tolcapone and Entecapone block L-dopa metabolism by inhibiting COMT.


Enteapone only acts peripherally while tolcapone acts in both periphery and
CNS.

136. What is the mechanism of simethicone? *


0 из 1

A. Inhibits production of eicosanoid inflammatory mediators

B. Activates opioid receptors in the enteric nervous system

C. Changes surface tension of gas bubbles, allowing easier passage of gas

D. 5HT3- receptor antagonist


E. Soluble but non-absorbable osmotic laxative

Правильный ответ

C. Changes surface tension of gas bubbles, allowing easier passage of gas

Комментарий

Simethicone changes surface tesnion of gas bubbles causing collapse of


foam and allowing easier passage of gas. A is mesalamine, B is
diphenoxylate, D is ondansetron and E is lactulose.

137. Which anticancer drug works by interfering with the mitotic spindle
and preventing microtubule disassembly into monomers? *
0 из 1

A. Cabazitaxel

B. Irinotecan

C. Teniposide

D. Vincristine

E. Carmustine

Правильный ответ

A. Cabazitaxel

Комментарий

Taxanes inhibit disassembly while vinca alkaloids (vincristine and vinblastine)


inhibit assembly.

138. A 36 year old male is brought to you for a 2 month history of


delusions, auditory hallucinations and disorganized catatonic behavior.
You diagnose him with possible schizophrenia. However, his wife reports
to you that the patient also had a history previous suicide attempts.
Which is the ideal antipsychotic for this patient? *
0 из 1

A. Clozapine

B. Olanzapine

C. Quetiapine

D. Ziprasidone

E. Aripiprazole

Правильный ответ
A. Clozapine

Комментарий

Clozapine is the only antipsychotic that also reduces the risk of suicide
among patients with schizophrenia.

139. Which anti-diabetic drug has side effects that can cause fluid
retention and worsen congestive heart failure?
1 из 1

A. Sulfonyurea

B. Meglitinide

C. Biguanide

D. SGLT2-inhibitor

E. Thiazolidinedione

Комментарий

Thiazolidinediones (pioglitazone, rosiglitazone and troglitazone) are


associated with fluid retention, anemia, weight gain, CHF, bone fractures,
cardiac events and increased risk of MI.

140. Ibutilide belongs to what class of antiarrhytmic drug?


0 из 1

A. Class 1A

B. Class 1B

C. Class 2

D. Class 3

E. Class 4

Правильный ответ

D. Class 3

Комментарий

Ibutilide is a Class 3 antiarrhythmic acting on K channels.

Surgery (141-160)
Баллов: 9 из 20.
141. A 23 year old male suffers from acute onset R lower quadrant pain.
He underwent an appendectomy. How will you classify this surgical
operation's wound class ? *
0 из 1

A. Class I

B. Class II

C. Class III

D. Class IV

E. Class V

Правильный ответ

B. Class II

Комментарий

Appendectomies are classified as clean-contaminated wounds.

142. Given this classification in #141, what are the estimated infection
rates for the patient? *
0 из 1

A. 0.5-1%

B. 1-2%

C. 2.1-14%

D. 15-30%

E. >30%

Правильный ответ

C. 2.1-14%

Комментарий

Class II wounds gace a 2.1-14% infection rate.

143. You are the resident on duty when a 40 year old male is brought to
the Emergency Room after a motorcycle crash. He looks mildly anxious,
with a HR of 125, a BP of 80/60 and an RR of 25. What is his estimated
blood loss in ml? *
0 из 1

A. <500
B. <750

C. 750-1,500

D. 1,500-2,000

E. >2,000

Правильный ответ

D. 1,500-2,000

Комментарий

First step is to classify the Class of Shock the patient is in. For HR>120 and
decreased BP, he is classified as Class III, with approximately 31-40% blood
loss or 1500,2000 ml.

144. A 40 year old male suffered from a gunshot wound on rhe RUQ of
his abdomen and is brought to the ER. He is conscious, coherent and
hemodynamically stable. What is your next step ? *
0 из 1

A. Do a quick FAST

B. Do an abdominal CT scan

C. Proceed with DPL

D. Straight to OR for exploratory laparotomy

E. Observe the patient for now

Правильный ответ

B. Do an abdominal CT scan

Комментарий

See algorithm for penetrating abdominal injury.

145. A 19 year old male is brought to you for an open fracture. After
careful and detailed evaluation, you note a 6cm long wound with muscle
damage and some degree of comminution. Based on the Gustilo
classification, what antibiotic will you select for this patient? *
0 из 1

A. Cefazolin

B. Cefazolin + Penicillin

C. Cefazolin + Amniglycoside
D. Cefazolin + Aminoglycoside + Penicillin

E. Metronidazole

Правильный ответ

C. Cefazolin + Amniglycoside

Комментарий

For patients with Gustilo Grade II, a 1st generation


cephalosporin+aminoglycoside is the antibiotic of choice.

146. What age is a baseline screening mammogram recommended for


breast cancer risk management? *
1 из 1

A. 30

B. 35

C. 40

D. 45

E. 40

Комментарий

A baseline mammogram is recommended at 35 years old, but annual


mammograms begin at age 40.

147. This melanoma is associated with the worst prognosis due to the
prominence of the vertical growth phase? *
0 из 1

A. Superficial spreading

B. Lentigo Maligna

C. Acral Lentiginous

D. Desmoplastic

E. Nodular

Правильный ответ

E. Nodular
Комментарий

"Most common - superficial spreading


Worst prognosis - nodular".

148. This antireflux surgical procedure involves a 180 degree posterior


fundoplication lof the distal 4cm of the esophagus for GERD? *
0 из 1

A. Nissen fundoplication

B. Dor fundoplication

C. Toupet fundoplication

D. Belsey Mark IV

E. Hill posterior gastropexy

Правильный ответ

C. Toupet fundoplication

Комментарий

A Toupet fundoplication is a 180 degree posterior fundoplication of the


esophagus aimed to minimize the adverse effects of the Nissen 360 degree
fundoplication.

149. A 46 year old female complained of abdominal pain, anorexia,


nausea, vomiting and weight loss not relieved by oral PPIs for a month.
On endoscopy, patient tested positive for Helicobacter pylori, and was
also found to have a low grade MALT lymphoma. What is your first line of
treatment? *
1 из 1

A. Eradication of H. pylori with PPI triple therapy

B. Surgery for excision of involved gastric tissue

C. Chemotherapy

D. EBRT

E. Continued endoscopic surveillance as most MALT lymphomas resolve

Комментарий

Similar boards concept. H. pylori eradication is the first line of treatment for
low grade gastric lymphoma as most are expected to resolve once the
inciting injury is treated.

150. A 28 year old female with a reported history of oral contraceptive


use consults you for a hepatic adenoma visualized on CT scan and MRI
with an estimated size of 8 cm. What is your management for this case?
*
0 из 1

A. Repeat imaging after 3 months

B. Observation only

C. Percutaneous aspiration

D. Sclerotherapy

E. Surgical resection

Правильный ответ

E. Surgical resection

Комментарий

Because hepatic adenomas are at risk of spontaneous rupture and


hemorrhage or malignant degeneration to a HCCA, these are always
resected in all cases.

151. This polyposis syndrome is associated with mutation of the PTEN


tumor suppressor gene with associated macrocephaly, trichilemmomas
and tumors of the thyroid, breast and uterus? *
0 из 1

A. Juvenile polyposis syndrome

B. Peutz-Jeghers syndrome

C. Cowden syndrome

D. Cronkite Canada syndrome

E. Bannayan-Riley-Ruvacalba syndrome

Правильный ответ

C. Cowden syndrome

Комментарий

Cowden syndrome Is an autosomal dominant condition with complete


pemnetrance by 2- years old, caused by a mutation at 10q22 (PEN tumor
suppressor gene) with asssociated macrocephaly, trichilemmomas and
malignant and benign neoplasms.

152. A patient who has an internal hemorrhoid that protrudes beyond the
anal verge but which spontaneously reduces has a? *
1 из 1
A. 1st degree hemorrhoid

B. 2nd degree hemorrhoid

C. 3rd degree hemorrhoid

D. 4th degree hemorrhoid

E. None of the above

Комментарий

"1st - does not go beyond anal verge


2nd - spontaneously reduces
3rd - needs manual reduction
4th - cannot be reduced, at risk for strangulation"

153. When considering acute limb ischemia from possible compartment


syndrome, what are the 6P's we look out for?? *
0 из 1

A. Pain, paresis, paresthesia, paralysis, pulselesness and poikilothermia

B. Pain, purple, paresthesia, paresis, pulselesness and poikilothermia

C. Pain, purple, paresthesia, paralysis, pulselesness and poikilothermia

D. Pain, pinkish, paresthesia, paralysis, pulselesness and pathology

E. Pain, pallor, paresthesia, paralysis, pulselesness and poikilothermia

Правильный ответ

E. Pain, pallor, paresthesia, paralysis, pulselesness and poikilothermia

Комментарий

The 6Ps are pain, pallor, paresthesia, paralysis, pulselesness and


poikilothermia.

154. This spinal cord syndrome is characterized by bilateral paresis of


upper>lower extremities, that is usually associated to hyperextension
injuries or cord compression? *
1 из 1

A. Central cord syndrome

B. Anterior cord syndrome

C. Posterior cord syndrome

D. Brown sequard syndrome


E. Cauda equina syndrome

Комментарий

Most common form of cervical spinal cord injury characterized by loss of


motion and sensation in arms and hands from trauma which causes damage
to the neck.

155. You are the resident on duty when one of the nurses reports to you
that they have been pricked by a needle as they were extracting blood
from an HIV patient. You advice the nurse to consult the hospital
infection control unit because estimated HIV transmission from a
needlestick injury is at around? *
1 из 1

A. 15%

B. 10%

C. 5%

D. 2%

E. <0.5%

Комментарий

Similar boards question.

156. A 33 year old male patient with a carcinoid tumor measuring 2.5cm
in the tip of the appendix should undergo what procedure? *
1 из 1

A. Observation only

B. Appendectomy

C. Resection of the cecum and terminal ileum

D. Wide resection of the mesoappendix and all appendiceal lymph nodes

E. Right hemicolectomy

Комментарий

Appendiceal carcinoid >2cm should undergo right hemicolectomy.

157. The thoracic portion of the esophagus receives its blood supply
from? *
1 из 1
A. Superior thyroid artery

B. Inferior thyroid artery

C. Bronchial arteries

D. Left gastric artery

E. Inferior phrenic artery

Комментарий

Similar board exam concept. B - cervical portion, C - thoracic portion, D and E


- abdominal portion.

158. According to the Surviving Sepsis guidelines, if your first line pressor
norepinephrine fails to achieve a target MAP >65, what is your second
line pressor? *
0 из 1

A. Dopamine

B. Vasopressin

C. Dobutamine

D. Milrinone

E. All of the above

Правильный ответ

B. Vasopressin

Комментарий

We suggest adding either vasopressin or epinephrine to norepinepjhrine with


the intent of raising the mean arterial pressure to target.

159. After shock and trauma, inflammatory cytokines are released. All of
the following are inflammatory cytokines except?
1 из 1

A. IL-1

B. IL-2

C. IL-6

D. IL-10
E. IL-12

Комментарий

Anti-inflammatory cytokines include IL-4, IL-10 and IL-13.

160. This is considered a true anatomic precursor of breast cancer?


1 из 1

A. LCIS

B. DCIS

C. Intraductal papilloma

D. Fibroadenoma

E. Sclerosing adenosis

Комментарий

DCIS is a true anatomic precursor, while LCIS only heralds an increased risk
for invasive cancer.

Internal Medicine (161-180)


Баллов: 10 из 20.

161. Gottron's papules are pathognomonic of what disease? *


0 из 1

A. Scleroderma

B. Mixed Connective Tissue Disease

C. Neurofibromatosis Type 1

D. Systemic Lupus Erythematosus

E. Dermatomyositis

Правильный ответ

E. Dermatomyositis

Комментарий

DM manifests with symmetric, proximal greater than distal weakness along


with a characteristic rash that includes the heliotrope rash (erythematous
discoloration of eyelids with periorbital edema), Gottron sign (erythematous
rash over the extensor surfaces of joints such as the knuckles, elbows, knees,
and ankles), Gottron papules (raised erythematous rash over knuckles) (Fig.
358-1), V-sign (rash on the sun-exposed anterior neck and chest), shawl sign
over the back of the neck and shoulders, nail bed telengiectasiae, and
subcuta- neous calcium deposits. (HPIM p. 2591, 20th edition).
162. A chronic cancer patient, bedbound for the past week, is referred to
you by the nurses. You suspect a diagnosis of pulmonary embolism, do a
quick Well's scoring for PE and get a score of 2. What test will you do
next? *
1 из 1

A. Chest CT with contrast

B. Chest X-ray

C. D-dimer

D. Pulmonary angiography

E. 12L-ECG

Комментарий

Similar boards question. The Well's Score for Pulmonary Embolism is used to
estimate the clinical likelihood of PE. A score of >4 means a HIGH CLINICAL
LIKELIHOOD (for which the intiial test of choice is an imaging test (a chest CT
with IV contrast is the principal imaging test for the diagnosis of PE), while a
score of 4 and below means a low clinical likelihood and a d-Dimer is sent.
(page 1911 HPIM 20th).

163. A 24 year-old female comes to you for hypogastric pain, dysuria and
a feeling of retention post-voiding, however she denies flank pain, fever
or chills. What is your drug of choice, and the dose? *
1 из 1

A. Nitrofurantoin 100mg QID x 3 days

B. TMP-SMX 160/800mg BID x 3 days

C. Ciprofloxacin 750mg/tab BID 14 days

D. Amoxicillin 1g TID x 5 days

E. Fosfomycin 3g single dose

Комментарий

Similar boards question. We were told during review that they don't ask for
the dose, however, during the September 2019 boards, we were asked about
the dose and duration for illnesses like UTI and CAP. First line for Acute
Uncomplicated Cystitis is Nitrofurantoin 100mg QID for 5 days or a single
dose of fosfomycin 3g. Cotrimoxazole (TMP-SMX) has high resistance and
should only be used with there is susceptibility to the drug.

164. During your ER duty, a 56 year old male comes in with severe,
crushing, crescendo-decrescendo chest pain radiating to the right
shoulder. A stat ECG was done revealing ST segment elevation in leads
V5 and V6. What wall is affected? *
1 из 1
A. Septal

B. Anterior

C. Lateral

D. Inferior

E. Anteroseptal

Комментарий

II, III, AVF - inferior wall; V1, V2 - septal, V3, V4 - anterior, V5, V6 - lateral; I
and avL - high lateral wall.

165. On physical examination he is noted to be slightly anxious and


restless.There was note of equal chest expansion, crackles on bilateral
bases and an S3 gallop of cardiac examination. What is his Killip score? *
0 из 1

A. Killip 1

B. Killip 2

C. Killip 3

D. Killip 4

E. Killip 5

Правильный ответ

B. Killip 2

Комментарий

Killip scoring is used as an independent predictor of early death among MI


patients. Killip 1 - individuals with no clinical sigs of heart failure; Killip 2 -
bibasal rales, S3 gallop and jugular venous distention; Killip 3 - pulmonary
edema, rales extending to >50% of your lung fields; Killip 4 - cardiogenic
shock.

166. An 86 year old man came in with hyponatremia as low as 123


mmol/L and dehydration from poor oral intake and a consideration of
SIADH. He was given a fast drip of liters of plain saline and after 12
hours, was found to be confused and with nausea and vomiting; with a
repeat serum sodium of 136mmol/L. What was this patient at risk for? *
1 из 1

A. Central pontine myelinosis

B. Diabetes insipidus
C. Cerebral edema

D. Increased ICP from fluid overload

E. Uremia

Комментарий

Overly rapid correction from a low sodium to a high sodium level puts
patients at risk for central pontine myelinosis or osmotic demyelination
syndrome. On the other hand, overly rapid correction of hypernatremia is
associated with cerebral edema.

167. A 24 year old female consults you with a finding of an anterior


mediatinal mass found on chest examination, with accompanying B
symptoms (fever, night sweats, weight loss). You suspect a possible
lymphoma. What is the most common aggressive histologic subtype of
Non-Hodgkin's Lymphoma? *
0 из 1

A. Diffuse Large B Cell

B. Follicular

C. Burkitt

D. Mantle Cell

E. Cutaneous T-cell

Правильный ответ

A. Diffuse Large B Cell

Комментарий

Diffuse large B-cell lymphoma (DLBCL) is the most common histologic


subtype of NHL diagnosed, representing about one-third of all cases. Among
the aggressive diseases, the most common are NHL and DLBCL; and the
most rapidly prolific are NHL and BL. FL is the second most common NHL and
the most common indolent NHL. (HPIM p. 774, 20th edition).

168. A 36 year old female was brought to the emergency room with a
fever of 38.6 degrees, a heart rate of 98, a respiratory rate of 21 and a
BP of 90/60. She on interviewing, she was awake, oriented and responds
to questions. What is her qSOFA score? *
0 из 1

A. 0

B. 1

C. 2

D. 3
E. 4

Правильный ответ

B. 1

Комментарий

The qSOFA scoring scores ranges from 0-3 and gives 1 point each for
GCS<15, RR>=22 and SBP=<100.

169. In which clinical samples can a diagnosis of gonococcal infection be


rapidly done? *
0 из 1

A. Gram stain of cervical smears of women

B. Gram stain of rectal swabs in children

C. Gram stain of urethral exudates of men

D. Gram stain of urethral exudates of women

E. All of the above

Правильный ответ

C. Gram stain of urethral exudates of men

Комментарий

A rapid diagnosis of gonococcal infection in men may be obtained by Gram’s


staining of urethral exudates. Unfortunately, the presence or absence of
gram- negative intracellular monococci or diplococci in cervical smears does
not accurately predict which patients have gonorrhea, and the diagnosis in
this set- ting should be made by culture or another suitable nonculture
diagnostic method. HPIM 1126-1127, 20th edition.

170. Which factor is implicated in angiosarcoma of the liver? *


1 из 1

A. Iron

B. Paracetamol

C. Alcohol

D. Schistosomia

E. Vinyl chloride

Комментарий
Angiosarcoma of the liver is associated with exposure to vinyl chloride,
arsenic or thorium dioxide.

171. In prolonged fasting states, what is the first defense against


hypoglycemia? *
0 из 1

A. Increased glucagon secretion

B. Increased epinephrine secretion

C. Increased cortisol secretion

D. Decreased glucagon secretion

E. Decreased insulin secretion

Правильный ответ

E. Decreased insulin secretion

Комментарий

Thus, a decrease in insulin secretion is the first defense against


hypoglycemia. HPIM p. 2883 20th edition.

172. All would show transudative pleural effusions EXCEPT *


1 из 1

A. Uremia

B. Nephrotic Syndrome

C. Super Vena Cava Obstruction

D. Cirrhosis

E. Congestive Heart Failure

Комментарий

Causes of transudative effusions include Congestive heart failure, Cirrhosis,


Nephrotic syndrome, Peritoneal dialysis, Superior vena cava obstruction,
Myxedema, Urinothorax (HPIM p. 2008, 20th edition).

173. Your 46 year old HIV patient non-compliant to his antiretroviral


treatment is on follow up at the OPD, and now complains of crampy
abdominal pain, fever, and a non-bloody diarrhea for one week now. An
endoscopy was done revealing multiple mucosal ulcerations and
intranuclear and cytoplasmic inclusion bodies on biopsy. What is the
cause for his diarrhea? *
0 из 1
A. Salmonella

B. Giardia

C. Campylobacter

D. Cryptosporidia

E. CMV

Правильный ответ

E. CMV

Комментарий

CMV colitis presents as diarrhea, abdominal pain, weight loss, and anorexia.
The diarrhea is usually nonbloody, and the diagnosis is achieved through
endoscopy and biopsy. Multiple mucosal ulcerations are seen at endoscopy,
and biopsies reveal characteristic intranuclear and cytoplasmic inclusion
bodies. Secondary bacteremias may result as a consequence of thinning of
the bowel wall. Treatment is with either ganciclovir or foscarnet for 3–6
weeks. Infections with enteric pathogens such as Salmonella, Shigella, and
Campylobacter are more common in men who have sex with men and are
often more severe and more apt to relapse in patients with HIV infection.
Patients usually present with crampy abdominal pain, fever, and bloody
diarrhea.

174. Among patients with limited cutaneous systemic sclerosis which


antibody will be positive, but negative for dcSSc (diffuse cutaneous
systemic sclerosis)? *
0 из 1

A. Anti-scl70

B. Anti-DNA Topoisomerase I

C. Anti-U1RNP

D. Anti-centromere antibody

E. Anti-RNA polymerase III

Правильный ответ

D. Anti-centromere antibody

Комментарий

Table 353-4 of HPIM 20th edition highlights the major systemic sclerosis
antibodies. Anti-scl70/antiDNA Topoisomerase I and Anti-RNA polymerase III
all are positive for dcSSC; anti-U1 RNP is positive for MCTD, while anti-
centromere antibody is for lcSSC.

175. Which subtype of melanoma is the most common? *


1 из 1
A. Acral lentiginous

B. Lentigo maligna

C. Nodular

D. Superficial spreading

E. None of the above.

Комментарий

Superficial spreading melanoma is the most common type.

176. A 55 year old male who works for processing in high tech industries
presents to you with persistent cough, a chest x-ray showing nodules
along septal lines, and a pulmonary function test showing a restrictive
ventilatory defect. Which inorganic dust is likely associated with the
patient's exposure? *
1 из 1

A. Asbestos

B. Berrylium

C. Coal dust

D. Silica

E. Cotton dust

Комментарий

Table 283.1 of HPIM 20th edition shows occupational exposures and


associated respiratory conditions. A job processing alloys at high tech
industries is associated with berrylium exposure.

177. All of the following increase the risk of Systemic Lupus


Erythematosus EXCEPT *
0 из 1

A. Drinking alcohol

B. Tobacco smoking

C. Crystalline silica exposure

D. Femaleness

E. UV light
Правильный ответ

A. Drinking alcohol

Комментарий

Drinking alcohol (2 glasses of wine a week or 1⁄2 of an alcoholic drink daily)


reduces the risk of SLE. HPIM 20th edition, p. 2517

178. A 26 year old male consults you for a 1 week history of fever, chills
and cough with associated greening yellowish sputum. Because his vital
signs were within normal and he does not have any unstable comorbids,
you decide to treat him for Community Aquired Pneumonia, Low Risk.
Which drug will you give? *
0 из 1

A. Amoxicillin 500mg TID

B. Clarithromycin 250mg OD

C. Co-amoxiclav 500mg BID

D. Azithromycin 100mg OD

E. Cefuroxime 500mg BID

Правильный ответ

E. Cefuroxime 500mg BID

Комментарий

Similar boards question. We were told during review that they don't ask for
the dose, however, during the September 2019 boards, we were asked about
the dose and duration for illnesses like UTI and CAP. Choices for CAP-LR
according to the CAP PCPG include amoxicillin 1g TID, extended macrolides
(azithromycin 500mg OD, clarithromycin 500mg BID), coamoxiclav 1g BID,
cefuroxime 500mg BID.

179. The patient returns, with his symptoms worse than before, was
subsequenlty admitted, and develops dyspnea, distress and
desaturations. Which among the following antibiotics will you NOT use if
you plan on covering for Pseudomonas aeruginosa? *
1 из 1

A. Meropenem

B. Cefepime

C. Ceftolozane

D. Ceftriaxone

E. Piperacillin-tazobactam
Комментарий

Ceftriaxone is a third generation cephalosporin with no coverage for


Pseudomonas aeruginosa.

180. A 60 year old chronic uncontrolled diabetic man who has undergone
bilateral below the knee amputations for neuroschemic foot ulcers now
comes to you for occasional episodes of chest pain on greater than
ordinary exertion. Your initial 12L-ECG and 2D-Echo findings were
unremarkable with no confounding features noted. What is your next
step if the diagnosis of ischemic heart disease is uncertain? *
1 из 1

A. Treadmill Stress Test

B. Pharmacologic Stress Test

C. Repeat 2D-Echo

D. Repeat 12L-ECG

E. Send for cardiac biomarkers

Комментарий

Similar boards question. Figure 267.3 of Harrisons shows an algorithm for


identifying patients who should be referred for stress testing and the decision
pathway for determining whether a standard treadmill exercise with
electrocardiogram (ECG) monitoring alone is adequate. A specialized
imaging study is necessary if the patient cannot exercise adequately
(pharmacologic challenge is given) or if there are confounding features on
the resting ECG (symptom-limited treadmill exercise may be used to stress
the coronary circulation).

OB-GYNE (180-200)
Баллов: 15 из 20.

181. A 24 year old whose LMP was 4 weeks ago now comes to you for
one day history of abdominal pain, breast tenderness and vomiting but
no vaginal bleeding or pain on urination. Her urine pregnancy test was
positive. On physical examination, vital signs were normal, however there
was note of mild hypogastric pain, a normal sized uterus and a closed
cervix with no bleeding. What is your next step? *
0 из 1

A. CBC

B. TV-UTZ

C. Urinalysis

D. Quantitative HCG level

E. Repeat urine pregnancy test

Правильный ответ

D. Quantitative HCG level


Комментарий

Similar board exam concept. A quantitative beta hcg level will approximate
the gestational age of the pregnancy and give a clue on expected ultrasound
findings because for values <1,500 we do not expect to detect at intrauterine
pregnancy yet, as it is below the discriminatory zone.

182. After your initial workup one of the tests you ordered, the
quantitative HCG level came back at 1,200 mIU/ml. What do you advice
your patient *
0 из 1

A. Schedule a consult with her OB-GYN of choice

B. Come back after 2 days for repeat quantitative B-HCG measurement

C. Proceed with methotrexate therapy

D. Undergo laparoscopy for evacuation of ectopic pregnancy

E. Proceed with ultrasonography

Правильный ответ

B. Come back after 2 days for repeat quantitative B-HCG measurement

Комментарий

This patient's HCG levels are still below the discriminatory zone and we do
not expect to detect any IUP during ultrasound at this point, so we repeat B-
HCG after 48 hours.

183. She comes back after 2 days with a B-HCG of 2500 mIU/ml. An
ultrasound done showed an intrauterine gestational sac with yolk sac
consistent with a 5 week pregnancy. Your patient added that her LMP
was on September 3, 2019. According to Naegele's rule, when is her
expected date of delivery? *
1 из 1

A. June 27, 2020

B. July 3, 2020

C. July 10, 2020

D. July 17, 2020

E. August 3, 2020

Комментарий

Naegele's Rule estimates date of delivery by subtracting 3 months and


adding 7 days to the patient's LMP.
184. What is the preferred parameter for pediatric aging in the first
trimester? *
1 из 1

A. Fundic height

B. Crown to rump length

C. Biparietal diameter

D. Femoral Length

E. Abdominal Circumference

Комментарий

"Fundic Height reliable only from 20-34 weeks.


Crown to rump length 4-12 weeks
Biparietal diameter 12-20 weeks
Femur length >20 weeks"

185. What is the expected amnionicity/chorionicity of a twin pregnancy if


cleavage occurs at Day 6? *
0 из 1

A. Dichorionic diamnionic

Dichorionic monoamniotic

monochorionic diamniotic

monochorionic monoamnionic

Conjoined twins

Правильный ответ

monochorionic diamniotic

Комментарий

DAY 1-3 DIchorionic, diamnionic


Day 4-8 Monochoronic, diamnionic
Day 8-13 monochorionic, monoamnionic
Day 14 beyond- Conjoined

186. Which of the following are correctly matched? *


1 из 1

A. Early deceleration: uteroplacental insufficiency

B. Late deceleration: head compression


C. Variable deceleration: cord compression

D. None of the above

E. All of the above

Комментарий

Early deceleration: head compression during a contraction


Variable deceleration: umbilical cord compression
Late deceleration: uteroplacental insufficiency

187. While in labor, a mother was noted to have late decelerations on her
tocogram. What potential interventions will you do to improve fetal
oxygenation and uteroplacental blood flow? *
1 из 1

A. Place mother in left lateral decubitus

B. Hook her to O2 support.

C. Administer IV fluid bolus

D. Discontinue oxytocin and prostaglandins

E. All of the above

Комментарий

Similar board exam concept. Table 24.3 of Williams shows resuscitative


measures for Cat II and Cat III tracings

188. This layer overlies the enlarging blastocyst and separates the
conceptus from the rest of the uterine cavity. *
1 из 1

A. Decidua basalis

B. Decidua parietalis

C. Decidua vera

D. Decidua capsularis

E. Nitabuch Layer

Комментарий

"Decidua basalis - layer directly underneath blastocyst implantation


Decidua parietalis - layer lining the remainder of the uterus
Decidua vera - layer formed by apposition of decidua capsularis and
parietalis that obliterates the uterine cavity
Nitabuch layer - zone where invading trophoblasts meet the decidua basalis,
if defective makes placenta accreta more likely"

189. This Leopold's maneuver is done with the examiner facing the
maternal head and palpating both sides of the uterus to determine which
side the hard, resistant, convex fetal back is located? *
1 из 1

A. Fundal grip

B. Umbilical grip

C. Pawlik's grip

D. Pelvic grip

E. None of the above

Комментарий

Similar board exam concept. Know the other names for maneuvers 1, 2, 3,
and 4.

190. Which is true regarding the management of Meig's syndrome? *


1 из 1

A. The ovarian mass will need chemotherapeutic drugs

B. The ovarian mass will recur if not treated with radiotherapy

C. Resection of the fibroma will cause resolution of the pleural effusion and
ascites

D. An elevated CA-125 is always associated with an ovarian malignancy

E. All of the above

Комментарий

1) an ovarian mass with pleural and abdominal effusion not always


represents an advanced malignancy; 2) even if elevated Ca 125 value is
usually associated to a ovarian malignancy, there are some benign lesions in
which we observed elevated level of this marker 3) the removal of the
ovarian mass is the only resolutive treatment for these patients.

191. A mother brings her child to your clinic because she has had no
menstrual period yet at 17 years old. On examination, you note bilateral
breasts present, absence of pubic and axillary hair, palpable masses at
the labia majora and groin, and a vagina that ends in a blind pouch?
There was no noted uterus on TV-UTZ. What is your impression? *
1 из 1

A. Transverse Vaginal Septum


B. Mayer-Rokitansky-Kuster-Hauser syndrome

C. 17-alpha hydroxylase enzyme deficiency

D. Androgen insensitivity syndrome

E. Turrner syndrome

Комментарий

AIS is due to a dysfunction of absence of a testosterone receptor leading to a


phenotypic female with 46XY features.

192. What may be advised to the patient? *


1 из 1

A. She will be unable to reproduce

B. A neovagina may be created for sexual functioning

C. The testicular masses will have to be removed surgically

D. She is at risk for testicular cancer in the future

E. All of the above

Комментарий

Patients with angrogen insensitivity syndrome/testicular feminization have


46XY chromosomes but are phenotypically female. Testes may be
undescended or may have migrated down to the labia majora. Diminished
sensitivity to testosterone leads to absence of pubic and axillary hair.
Because estrogen is produced, they usually have breast, and a vagina that
ends in a blind pouch.

193. A 36 year old pregnant G3P2 comes to you complaining of vulvar


itching and on examination, you note a thick, white, curdlike discharge on
the vaginal walls. Wet prep showed a PH of 4.0, and pseudohyphae. What
treatment will you give her? *
1 из 1

A. Metronidazole 500mg/tab BID

B. Clindamycin 300mg/cap BID

C. Azithromycin 1g single dose

D. Clotrimazole cream

E. None; this is normal physiologic discharge


Комментарий

Vulvovaginal candidiasis is treated with azole creams. Metronidazole may be


used for bacterial vaginosis and trichomonas, clindamycin may ba used for
bacterial vaginosis. Azithromycin is used to treat mucopurulent cervicitis and
gonorrhea (with ceftriaxone 250mg IM injection).

194. A concerned mother brings her 14 year old daughter to your clinic
worried that her daughter has not started menstruation yet. On
examination, you note unremarkable physical exmaination findings and
an SMR of 3? What will you advice the mother and her daughter? *
1 из 1

A. Order a transrectal ultrasound

B. Send for karyotyping studies

C. Measure serum LH, FSH

D. Request for cranial MRI

E. Reassure mother and observe

Комментарий

Similar board exam concept. Primary amenorrhea is absence of menses at


16 years old in the presence of secondary sexual characteristics. In her case,
the patient is SMR 3, and is expected to have her menarche soon.

195. A 25 year old primigravid at 8 weeks AOG consults you for a one day
history of hypogastric pain,, and vaginal spotting, which progressed to
passage of meaty material today, prompting her to visir your clinic. On
your speculum examination, her cervix was closed and there was no
bleeding per os You did an ultrasound which showed an empty uterus.
What is your next step? *
0 из 1

A. Observe patient for now

B. Get consent to do a dilatiion and curretage

C. Start her on tranexamic acid capsules 500mg/cap TID

D. Start her on co-amoxiclav IV antibiotics

E. Administer oxytocin IV

Правильный ответ

A. Observe patient for now

Комментарий

This is the case of a complete abortion, and observation os the only thing
warranted at this point to watch out for bleeding, signs of sepsis or infection
etc.
196. A 25 year old female consulted your clinic for her annual
gynecologic check-up. You relay to her that the findings showed cervical
dysplasia confined to the lower two-thirds of the epithelium. How would
you classify these findings? *
1 из 1

A. CIN I

B. CIN II

C. CIN III

D. CIS

E. Invasive cervical cancer

Комментарий

CIN I cellular dysplasia in the basal third of the epithelium, CIN II confined to
basal 2/3, CIN III more than 2/3 of the epithelium, CIS if entire epithelium is
abnormal, invasive cervical cancer if abnormalities invade through basement
membrane into the cervical stroma.

197. Which of the following is an expected finding in post menopausal


women? *
1 из 1

A. Increasing inhibin levels

B. Increasing sex hormone binding globulin levels

C. Increasing FSH levels

D. Increasing estradiol levels

E. Increasing testosterone levels

Комментарий

In menopause, FSH levels elevate in order to promote follicular development.


(10-20x), LH increase 3x, testosterone decreases, SHBG decreases, estradiol
markedly declines.

198. Which of the following is an initial endocrinologic change occuring


during the onset of menopause *
1 из 1

A. Elevated Anti-Mullerian Hormone

B. Elevated Ovarian Inhibin B


C. Elevated FSH

D. Elevated Estrogen

E. Elevated Progesterone

Комментарий

The initial endocrinologic change signaling the onset of menopause is


decreased AMH and ovarian inhibin-B production accompanied by an
increase in FSH.

199. After ovulation and before fertilization, the secondary oocyte is


arrested at which stage? *
1 из 1

A. Prophase I

B. Prophase II

C. Metaphase II

D. Anaphase II

E. Telophase II

Комментарий

The primary oocyte is arrested in prophase from the 5th fetal month until
onset of puberty. During puberty, the first meotic division is completed. After
ovulation, the secondary oocyte is arrested in metaphase II until fertilization.
After fertilization, the 2nd meiotic division is completed.

200. Which type of uterine myoma is associated with heavy and


prolonged bleeding?
0 из 1

A. Subserosal

B. Submucosal

C. Intramural

D. Cervical

E. None of the above

Правильный ответ

B. Submucosal

Комментарий
Submucosal myomas are associated with heavy and prolonged bleeding

Pediatrics (200-220)
Баллов: 9 из 20.

201. An infant was delivered to a 35 year old multipara and you assisted
the pediatrician. The infant was noted to have a heart rate of 120, a pink
body but blue extremities, flexed arms and legs and vigorous crying.
What is the APGAR score of this infant? *
0 из 1

A. 6

B. 7

C. 8

D. 9

E. 10

Правильный ответ

C. 8

Комментарий

Arms and legs flexed 1, Heart rate 120 2, Vigorous cry 2, Reflex irritability 2,
Blue extremities 1
Topnotch page 3 Handout

202. A newborn 14 day old infant was referred to you for episodes of
vomiting, lethargy and abnormal laboratory results. He was found to
have a serum sodium of 130, a serum potassium of 5.6 with episodes of
low blood sugar. What is the most commonly deficient substance in this
case? *
0 из 1

A. Thyroid hormone

B. 21-hydroxylase enzyme

C. Galactose-1-phosphate uridyltransferase deficiency

D. Phenylalanine hydroxylase

E. Branched chain alpha ketoacid dehydrogenase complex

Правильный ответ

B. 21-hydroxylase enzyme

Комментарий

Absence of 21-hydroxylase enzyme in Congenital Adrenal Hyperplasia in


infants causes failure to thrive, vomiting and findings of hyponatremia,
hyperkalemia, hypoglycemia due to a deficiency in cortisol and aldosterone.
This will need glucocorticoid replacement with hydrocortisone and
mineralocorticoid replacement with fludrocortisone.

203. Your intern refers to you a newborn infant who on monitoring was
noted to have frothing and bubbling at the mouth, and noted cough and
cyanosis on feeding. What is the most common type of this lesion you are
thinking the infant has? *
0 из 1

A. Esophageal atresia with double tracheoesophageal fistula (TEF)

B. Esophageal atresia with proximal TEF

C. Isolated TEF

D. Isolated EA

E. Esophageal atresia with distal TEF

Правильный ответ

E. Esophageal atresia with distal TEF

Комментарий

The most common type of TEF is an EA with distal TEF for which surgical
ligation of the TEF and a primary end to end anastomosis is done.

204. A newborm infant was found to have Total Anomalous Pulmonary


Venous Return. Which type of TAPVR is the most common? *
1 из 1

A. Supracardiac

B. Cardiac

C. Infracardiac

D. Mixed

E. None of the above

Комментарий

The supracardiac type is the most common, and it accounts for 44% of cases,
followed by the infracardiac (26%), cardiac (21%), and mixed type (9%). In
the supracardial type, the pulmonary veins drain through a vertical vein into
the left brachiocephalic vein but rarely drain to the superior vena cava, left
superior vena cava, or to the azygous system . In the cardiac type, the
pulmonary veins drain to the coronary sinus or directly to the right atrium In
the infracardiac type, the second most common, the drainage is located
below the diaphragm, and may be either to a systemic vein such as inferior
vena cava, hepatic veins or azygos system, or to the portal venous system.
205. Which of the following does NOT classify as pathologic jaundice? *
1 из 1

A. Onset of jaundice within the first 24 hours of life

B. A rise in serum bilirubin of >5mg/dl/day

C. A serum bilirubin level of >12mg/dl

D. Jaundice persisting after >14 days

E. Direct bilirubin of >1mg/dl at any time

Комментарий

A direct bilirubin level of 1 mg/dl may be normal, but levels >2mg/dl at an


time warrants investigation for pathologic jaundice.

206. This vaccine is associated with seizures and hypotonic episodes as


its adverse effect? *
0 из 1

A. Vitamin B1

B. Vitamin B2

C. Vitamin C

D. Vitamin D

E. Vitamin E

Правильный ответ

C. Vitamin C

Комментарий

Vitamin C deficiency is associated with low grade fever, generalized


tenderness of the legs, a scorbutic rosary, mucous membrane swellings,
swollen joints, purpura, ecchymoses and poor wound healing, with
hyperkeratosis of hair follicles and a corkscre hair.

207. This vaccine is associated with seizures and hypotonic episodes as


its adverse effect? *
1 из 1

A. Diphtheria

B. Pertussis

C. Tetanus
D. Measles

E. Varicella

Комментарий

The pertussis vaccine (cellular) component is associated with neuroparalytic


effects so for adults, it may be given as acellular pertussis (DtaP), or not
included (Td vaccine).

208. This reflex is expected to persist throughout a person's life? *


1 из 1

A. Palmar grasp

B. Rooting

C. Moro

D. Tonic Neck

E. Parachute

Комментарий

The parachute reflex remains throughout life and is a prerequisite to walking.

209. A 1 month old girl was referred to you for a one week history of
non-bilous vomiting, irritability and weakness. Imaging done in the
previous hospital showed (+) shoulder sign and (+) double tract sign.
What is your impression? *
1 из 1

A. Intussuception

B. Esophageal atresia

C. Pyloric stenosis

D. Duodenal atresia

E. Hirschprung's disease

Комментарий

Pyloric stenosis is confirmed via ultrasound with pyloric thickness >4mm, and
length>14mm, and a bulge of the pyloric muscle into the antrum (shoulder
sign) and streaks of barium in the narrow channel (double tract sign).
210. An 10-month old male was diagnosed with UTI on the basis of
symptoms of fever, an ill appearance and costovertebral angle
tenderness, along with positive cultures and was treated with antibiotics
for 1 week. After clinical improvement, what is your next step? *
0 из 1

A. Repeat urine culture

B. Abdominal radiography

C. Renal and bladder ultrasonography

D. Voiding Cystourethrogram

E. None of the above

Правильный ответ

C. Renal and bladder ultrasonography

Комментарий

Pyloric stenosis is confirmed via ultrasound with pyloric thickness >4mm, and
length>14mm, and a bulge of the pyloric muscle into the antrum (shoulder
sign) and streaks of barium in the narrow channel (double tract sign).

211. Findings of a microcytic anemia with low serum iron, low TIBC and
high ferritin is indicative of? *
0 из 1

A. Iron Deficiency

B. Lead poisoning

C. Thalassemia

D. Chronic Disease

E. None of the above

Правильный ответ

D. Chronic Disease

Комментарий

Topnotch Pedia p. 38

212. Polyostotic fibrous dysplasia, café au lait macules and precocious


puberty is a finding in: *
1 из 1

A. Neurofibromatosis 1
B. Neurofibromatosis 2

C. Tuberous Sclerosis

D. McCune-Albright Syndrome

E. Russell-Silver Syndrome

Комментарий

Topnotch Pedia p.51

213. A young mother consults you because she is concerned of her 2


month old baby has been having poor feeding and persistent nasal
discharge/snuffles. What is the suspected microbiologic agent causing
this? *
1 из 1

A. Streptococcus pneumonia

B. Hemophilus influenzae

C. Chlamydia trachomatis

D. Treponema pallidum

E. Parainfluenza virus

Комментарий

Persistent rhinorrhea in the first three months of life may be "syphilitic


rhinitis", which appears similar to the rhinitis of the common cold, except it is
more severe, lasts longer, often involves bloody rhinorrhea caused by
Treponema pallidum.

214. A 6 year old female presents to you with a high grade fever
associated with brassy cough with copious purulent secretions. A lateral
x-ray view showed a ragged tracheal column. What is the treatment for
this? *
0 из 1

A. Short acting beta agonist and inhaled corticosteroids

B. Nebulized racemic epinephrine and Dexamthasone 0.6mg/kg single dose

C. Aqueous Pen G 150,000 U/kg/day q6hours

D. Vancomycin and a 3rd generation cephalosporin

E. Supportive treatment only


Правильный ответ

D. Vancomycin and a 3rd generation cephalosporin

Комментарий

This patient has bacterial tracheitis often caused by Staphylococcus aureus


(or MRSA, Strep. Pneumoniae) which is characterized with high fever, toxic
looking child with mucosal swelling at the cricoid cartilage and copious thick
purulent secretions. The ragged tracheal column is indicative of
pseudomembrane detachment. This is treated with vancomycin or
clindamycin AND a 3rd generation cephalosporin.

215. A known HIV infected mother who was non-compliant to her ARTs
brought her one year old infant to your clinic. The infant was noted to
have encephalopathy, wasting and regression of previously achieved
developmental milestones. Among children with such symptoms >75%
die before what age? *
0 из 1

A. 1

B. 2

C. 3

D. 4

E. 5

Правильный ответ

C. 3

Комментарий

Nelsons page 1665. Children with opportunistic infections (Pneumocystis


pneumonia, MAC), encephalopathy, regressing developmental milestones
and wasting syndromes have rhe worst prognosis with 75% dying before 3
years of age.

216. We begin to suspect for an abnormality in sexual development if


there is no thelarche noted by what age? *
0 из 1

A. 10

B. 11

C. 12

D. 13

E. 14
Правильный ответ

D. 13

Комментарий

We suspect for an abnormality if there is no thelarche by 13; no menses,


hematocolpos or no vagina at 14; or with menses but other sexual
characteristics not present at 16 (Topnotch p.75).

217. The triad of thrombocytopenia, infections and eczema is associated


with what mutation? *
1 из 1

A. WASP mutation

B. 22q11.2 deletion

C. SH2D1A mutation

D. BTK mutation

E. None of the above

Комментарий

22q11.2 deletion - DiGeorge Syndrom


SH2D1A mutation - X-linked lymphoproliferative disorder or Duncan Disease
(fatal EBV)
BTK mutation - X-linked agammaglobulinemia
WASP mutation - Wiskott Aldrich Syndrome (thrombocytopenia, eczema and
infection; with prolonged bleeding from circumcision site and bloody
diarrhea during infancy).

218. What is the approximate age of an child who can draw a circle, has
20/30 vision and whose handedness has just been established? *
1 из 1

A. 3

B. 4

C. 5

D. 6

E. 7

Комментарий

3 - dry by night, rides a tricycle, knows name, age and sex, draws a circle.

219. Which is the first tooth to erupt in infants? *


0 из 1

A. Canine tooth

B. Upper premolars

C. Lower premolars

D. Maxillary central incisors

E. Mandibular central incisors

Правильный ответ

E. Mandibular central incisors

Комментарий

Tooth eruption usually behins at 6 months, with central mandibular incisors


erupting first

220. A 3 year old boy with no known comorbids was referred to your for
a low BP. On measurement you note his BP to be 80/60.You assure your
clerk that this blood pressure is within normal range because for his age,
the blood pressure will only be considered low if systolic BP drops below:
*
0 из 1

A. 54

B. 69

C. 72

D. 76

E. 84

Правильный ответ

D. 76

Комментарий

The formula for minimum SBP is 70+ (agex2). In this case, for a 6 year old, a
BP below 76mmHg is already considered low.

Preventive Medicine (220-240)


Баллов: 7 из 20.

221. The Expanded Breastfeeding Promotion Act of 2009 includes the


following EXCEPT *
1 из 1
A. Nursing employees shall granted break intervals in addition to the regular
time-off for meals to breastfeed or express milk.

B. Rooming-in as a national policy to encourage, protect and support the practice


of breastfeeding

C. Health and non-health facilities, establishments or institutions shall establish


lactation stations

D. Lactation stations may be located near the toilets in restrooms

E. Necessary equipment such as: lavatory for hand-washing refrigeration or


appropriate cooling facilities for storing expressed breastmilk; electrical outlets
for breast pumps; a small table and comfortable seats are needed

Комментарий

Under RA 7600 The lactation stations shall not be located in the toilet.

222. Your research group decides to do a cohort study. What is the main
outcome measure being derived from the data you will collect? *
0 из 1

A. Risk Ratio

B. Odds Ratio

C. PPV

D. Sensitivity

E. Confidence Interval

Правильный ответ

A. Risk Ratio

Комментарий

Cohort: Risk Ratio


Case Control: Odds Ratio

223. The other research group plans on doing a case-control study. They
will be computing for what outcome measure *
0 из 1

A. Risk Ratio

B. Odds Ratio

C. PPV
D. Specigicity

E. Confidence Interval

Правильный ответ

B. Odds Ratio

Комментарий

Cohort: Risk Ratio


Case Control: Odds Ratio

224. This model believes that if perceived barriers, benefits, self-efficacy,


and threat are successfully targeted, behavioral change is more likely
among patients *
0 из 1

A. Social Learning Model

B. Social Determinants of Health

C. Health Belief Model

D. Social Cognitive Model

E. Envorinmental Constructs Model

Правильный ответ

C. Health Belief Model

Комментарий

As one of the most widely applied theories of health behavior (Glanz &
Bishop, 2010), the Health Belief Model (HBM) posits that six constructs predict
health behavior: risk susceptibility, risk severity, benefits to action, barriers to
action, self-efficacy, and cues to action.

225. A study was conducted to investigate the causes of COPD. Patients


who had COPD was matched wth controls by age, sex, address and social
class. Among the two groups, the occurrence of biofuel exposure was
compared. This would classify as what type of study? *
0 из 1

A. Randomized Control Trial

B. Retrospective Cohort

C. Prospective Cohort

D. Case Control

E. None of the above


Правильный ответ

D. Case Control

Комментарий

Case Control - you start off with cases (those with COPD and those without),
match them with controls and compare their previous exposures

226. A study wanted to test the relationship between oral contraceptives


and breast cancer risk. 500 women were recruited, and answered a
survey regarding their use of oral contraceptives. Twenty years later,
these women were interviewed for incidence of histologically confirmed
breast cancer. This is an example of what type of study? *
1 из 1

A. Randomized Control Trial

B. Retrospective

C. Prospective Cohort

D. Case Control

E. None of the above

Комментарий

Cohort - you start with exposure (those exposed to OCPs or not) and follow
them to look for development of the outcome

227. Number of people who truly tested positive divided by the number
of people who actually have the disease? *
0 из 1

A. True positive

B. Positive predictive value

C. Negative predictive value

D. Specificity

E. Sensitivity

Правильный ответ

E. Sensitivity

Комментарий

Sensitivity is labelling positive thos who truly have the disease.


228. What is the probability of not having the condition, given a negative
test? *
0 из 1

A. False negative

B. Positive predictive value

C. Negative predictive value

D. Specificity

E. Sensitivity

Правильный ответ

C. Negative predictive value

Комментарий

Negative predictive value is computed by getting the number of people who


truly do not have the disease (TRUE NEGATIVES) divided by those who tested
negative.

229. For residency, you plan on conducting a randomized clinical trial on


the effect of febuxostat in the prevention of Tumor Lysis Syndrome
patients. You will need to attend and secure which in order to do so? *
0 из 1

A. Clinical Trials Conference Training

B. Ethucal Research Training

C. Good Clinical Practices Training

D. Skills for Researchers Training

E. None of the above

Правильный ответ

C. Good Clinical Practices Training

230. In a triple blind study, who among the following is blinded? *


1 из 1

A. Participants

B. Outcome assessors
C. Data analysts

D. All of the above

E. None of the above

Комментарий

All three are blinded in a triple blind study

231. An anesthetsiologist wanted to conduct a study investigating the


occurrence of post-operative nausea and vomiting among patients
treated with ondansetron, droperidol or placebo. Which statistical test
would be ideal in this case? *
0 из 1

A. Chi square test

B. Student t test

C. Regression analysis

D. Analysis of variance (ANOVA)

E. Multiple regression

Правильный ответ

A. Chi square test

Комментарий

For two qualitative variables (pesence/absence of postoperative nausea or


vomiting AND ondansetron vs droperidol vs placebo) a chi square test is used

232. In the previous question, what is your dependent variable? *


1 из 1

A. Presence of post operative nausea and vomiting

B. Anti-emetic drug used

C. Post operative patients

D. Surgery done to patients

E. None of the above


Комментарий

Dependent: Post operative nausea or vomiting


Independent: Medication used

233. In the management of dengue, for patients with profound and


prolonged shock the following resuscitative measures are used EXCEPT?
*
0 из 1

A. Colloids

B. Fresh Whole Blood

C. Fresh Frozen Plasma

D. Dpamine

E. None of the above

Правильный ответ

E. None of the above

Комментарий

Patients who remain in shock with low or normal haematocrit levels but have
excessive fluid accumulation, are most likely to have severe occult bleeding.
If the BP is low, a dopamine infusion should be started. Further infusion of
large volumes of intravenous fluids will lead to a poor outcome. Careful
transfusion of FWB, at least 10 ml/kg, should be initiated as soon as possible
at a rate titrated to a clinical response, blood gases and lactate. (WHO
Dengue guidelines)

234. Which component of the Family APGAR assesses satisfaction with


the way the respondent and his family spends time together? *
0 из 1

A. Adaptation

B. Participation

C. Growth

D. Affection

E. Resolve

Правильный ответ

E. Resolve

Комментарий

Adaptation - capability of the family to utilize and share resources


Partnership - ability to solve problems by communicating, Growth - freedom
to grow and change, Affection - intimacy and emotional interaction within the
family, Resolve - members satisfaction with the commitment made by the
members of the family

235. A 44 year old women presents to you with a right breast mass. On
initial biopsy, it was found to be a benign mass. However, your patient
decides to push through with surgical removal. On pathologic
examination of the excised surgical specimen, the sign out of the
pathologis was a breast adenocarcinoma. How will you describe the initial
biopsy done? *
1 из 1

A. True positive

B. True negative

C. False positive

D. False negative

E. None of the above

Комментарий

Similar board exam concept repeated 3x in varying formats and situations.


Because the initial biopsy was negative but the mass turned out malignant,
we consider it a falsely negative finding.

236. The National Newborn Hearing Screening Act includes all of the
following except? *
0 из 1

A. All infants born in hospitals in the Philippines shall be made to undergo


newborn hearing loss screening before discharge, unless the parents or legal
guardians of the newborn object to the screening

B. Infants who are not born in hospitals should be screened within one month
after birth.

C. In the event of a positive newborn hearing loss screening result, the newborn
shall undergo audiologic diagnostic evaluation in a timely manner to allow
appropriate follow-up, recall and referral for intervention before the age of six
months

D. A parent or legal guardian may refuse hearing loss screening on the grounds
of religious and/or cultural beliefs

E. Any healthcare practitioner who delivers, or assists in the delivery, of a


newborn in the Philippines shall prior to delivery, inform the parents or legal
guardian of the newborn of the availability, nature and benefits of hearing loss
screening among newborns or infants three (3) months old and below

Правильный ответ

B. Infants who are not born in hospitals should be screened within one month
after birth.

Комментарий
Infants who are not born in hospitals should be screened within the first three
(3) months after birth.

237. The Philippine Food Fortification Program includes regulation of the


following vitamins and minerals EXCEPT ? *
1 из 1

A. Vitamin K

B. Iodine

C. Vitamin A

D. Iron

E. None of the above

Комментарий

Philippine Food Fortification Program includes regulation of iodine, Vitamin A,


and Iron

238. A couple has five children. Their eldest daughter an engineer, got
married this year and is now living with her spouse in the US. The couple
is still supporting their youngest daughter aged 14. This family is in which
stage of the family life cycle:? *
0 из 1

A. Newly married couple

B. Unattached young adult

C. Family with young children

D. Family with adolescents

E. Launching family

Правильный ответ

E. Launching family

Комментарий

This stage starts when the first child leaves, and ends when the last child
leaves

239. This sampling method is at risk for bias if there is an underlying


periodicity in the population being sampled. *
0 из 1
A. Simple random sampling

B. Systematic sampling

C. Cluster sampling

D. Multistage sampling

E. Convenience sampling

Правильный ответ

B. Systematic sampling

Комментарий

The advantage of systematic sample is that it really fast and easy to


convenient when you already have a list of the units in your population. But
the disadvantage is that systematic sample might lead to bias. Suppose the
population is arranged in a pattern and if the periodicity of sampling might
exactly match the periodicity of that pattern. However, if we can assume that
the population list is randomly shuffled, then systematic sampling is
equivalent to simple random sample, where there is no bias.

240. This medication is associated with phocomelia when taken during


pregnancy? *
1 из 1

A. Lithium

B. Isotretinoin

C. Thalidomide

D. Warfarin

E. Methimazole

Комментарий

"Lithium - Ebstein's anomaly


Isotretinoin - multiple severe birth defect
Methimazole - aplasia cutis congenita
Warfarin - bone deformities, fetal hemorrhage, abortion, ophthalmic
abnormalities

Компания Google не имеет никакого отношения к этому контенту. - Условия использования -


Политика конфиденциальности

Вам также может понравиться